Этого треда уже нет.
Это копия, сохраненная 10 января 2021 года.

Скачать тред: только с превью, с превью и прикрепленными файлами.
Второй вариант может долго скачиваться. Файлы будут только в живых или недавно утонувших тредах. Подробнее

Если вам полезен архив М.Двача, пожертвуйте на оплату сервера.
Тред тупых вопросов №131 Planck edition 578517 В конец треда | Веб
Тред вопросов о жизни, Вселенной и всём таком.

Спрашиваем то, за что в других местах выдают путёвку в биореактор. Здесь анонимные учёные мирового уровня критически рассмотрят любые гениальные идеи и нарисованные в Paint схемы.

Предыдущий тут: >>567795 (OP)
https://2ch.hk/spc/res/567795.html (М)

Q: Можно быстрее?
A: Можно упасть в пузырь Альбукерке, NASA уже почти надула его.

Q: Я начитался охуительных историй про уфологию, че делать, нам жопа?
A: Да, тебе жопа, можешь сгонять в зогач или куда оттуда пошлют.

Q: Что будет с человеком в вакууме без скафандра / если он упадет на черную дыру / попробует ступить на поверхность газового гиганта/солнца?
A: Он умрёт.

Q: Почему бы не привязать ракету к воздушному шару или стартовать с горы?
A: Космос - это не как высоко, а как быстро, большая часть энергии ракеты уходит на разгон вбок.
Подробнее тут https://what-if.xkcd.com/58/ (английский) https://chtoes.li/orbital-speed/ (перевод)
2 578521
>>578517 (OP)
А Эра Водолея на картинке где?
3 578522
>>578517 (OP)
Ебанный стыд...
Во-первых, Алькубьерре.
Во-вторых, не упасть, а создавать вокруг корабля изнутри (иначе кина не будет).
В-третьих, НАСА искривляет пространство на десятимиллионную часть, контролируя это сверхточными интерферометрами, до самого варп-привода здесь - как до Антарктиды раком.
4 578523
Заготовленные учёные у меня закончились, может, кто-то другой в следующий раз перекатит? Только не снова с Джоселин, пожалуйста.
5 578525
>>578523
слышал сегодня у кипу торну 80 лет, мог бы с ним сделать
6 578542
>>578525
Торн был же вроде пару лет назад.

>>578523
Мессье, Галлей, Лаплас, Лагранж были?

О, я вроде знаю кого не было - Цвикки.

Продолжай перекатывать, у тебя хорошо получается. Я где-то до 80 или 90 катал, уже забыл как это делать, просто забуду это сделать и кто-то другой запилит.
7 578544
>>578542
Одна из этих картинок не такая как все.
Первая, на ней мужик не делает жест пальцами.
image.png240 Кб, 605x298
8 578545
>>578517 (OP)
Поясните про электрослабое взаимодействие, как это одновременно ЭМ и слабое?
И что значит "пустыня" на пикче?
9 578560
Правда что гарный хлопець Циолковский свою формулу придумал вычитав ее в британском учебнике 19 века?
10 578561
>>578545

>Поясните про электрослабое взаимодействие, как это одновременно ЭМ и слабое?


Это значит, что обычно слабое взаимодействие нихуя не похоже на электромагнитное — оно на много порядков слабее и действует только на очень коротких расстояниях, но чем выше энергии взаимодействующих частиц, тем более эти взаимодействия начинают походить друг на друга, и при очень высоких энергиях (сотни ГэВ) сливаются в одно.

Так-то фотон из электромагнитного взаимодействия и Z-бозон из слабого должны были в теории вести себя абсолютно идентично, но оказалось, что существует такая штука, как поле Хиггса, которое цепляет и тормозит W и Z-бозоны, придавая им тем самым нихуёвую массу, а с фотонами оно при этом никак не взаимодействует. Симметрия нарушается и из-за этого создается впечатление, что слабое и электромагнитное взаимодействие это разные вещи.

При энергиях выше нескольких сотен ГэВ механизм Хиггса перестает действовать, все бозоны снова становятся безмассовыми и оба взаимодействия начинают вести себя абсолютно одинаково. Сложный прикол, короче, на этом поле не одна нобелевка получена.

>И что значит "пустыня" на пикче?


https://en.wikipedia.org/wiki/Desert_(particle_physics)

Это так называется большой разрыв между самыми тяжелыми известными частицами (массой порядка 1012 эВ), самыми мощными взаимодействиями, которые мы можем экспериментально наблюдать — электрослабое объединение (тоже порядка 1012 эВ) и следующим этапом — теориями великого объединения, где энергии уже составляют порядка 1025 эВ. Т.е. это примерно 12-13 порядков, между которыми нет абсолютно нихуя интересного — ни новых частиц, ни новых взаимодействий.

Так пессимистично этот регион называется потому, что пересечь эти 13 порядков так же сложно, как пересечь пустыню пешком. Всё, что в физике частиц можно было открыть в обозримых окрестностях, мы уже открыли, и никто не даст бабла на новые ускорители частиц, которые будут в сто, тысячу или даже миллион раз мощнее существующих, т.к. на них нихуя нового открыто не будет, и в чем смысл их постройки, совершенно неясно. Ну а сразу прыгнуть на 13 порядков — такого даже в сказках не бывает.
10 578561
>>578545

>Поясните про электрослабое взаимодействие, как это одновременно ЭМ и слабое?


Это значит, что обычно слабое взаимодействие нихуя не похоже на электромагнитное — оно на много порядков слабее и действует только на очень коротких расстояниях, но чем выше энергии взаимодействующих частиц, тем более эти взаимодействия начинают походить друг на друга, и при очень высоких энергиях (сотни ГэВ) сливаются в одно.

Так-то фотон из электромагнитного взаимодействия и Z-бозон из слабого должны были в теории вести себя абсолютно идентично, но оказалось, что существует такая штука, как поле Хиггса, которое цепляет и тормозит W и Z-бозоны, придавая им тем самым нихуёвую массу, а с фотонами оно при этом никак не взаимодействует. Симметрия нарушается и из-за этого создается впечатление, что слабое и электромагнитное взаимодействие это разные вещи.

При энергиях выше нескольких сотен ГэВ механизм Хиггса перестает действовать, все бозоны снова становятся безмассовыми и оба взаимодействия начинают вести себя абсолютно одинаково. Сложный прикол, короче, на этом поле не одна нобелевка получена.

>И что значит "пустыня" на пикче?


https://en.wikipedia.org/wiki/Desert_(particle_physics)

Это так называется большой разрыв между самыми тяжелыми известными частицами (массой порядка 1012 эВ), самыми мощными взаимодействиями, которые мы можем экспериментально наблюдать — электрослабое объединение (тоже порядка 1012 эВ) и следующим этапом — теориями великого объединения, где энергии уже составляют порядка 1025 эВ. Т.е. это примерно 12-13 порядков, между которыми нет абсолютно нихуя интересного — ни новых частиц, ни новых взаимодействий.

Так пессимистично этот регион называется потому, что пересечь эти 13 порядков так же сложно, как пересечь пустыню пешком. Всё, что в физике частиц можно было открыть в обозримых окрестностях, мы уже открыли, и никто не даст бабла на новые ускорители частиц, которые будут в сто, тысячу или даже миллион раз мощнее существующих, т.к. на них нихуя нового открыто не будет, и в чем смысл их постройки, совершенно неясно. Ну а сразу прыгнуть на 13 порядков — такого даже в сказках не бывает.
11 578564
>>578561

>При энергиях выше нескольких сотен ГэВ механизм Хиггса перестает действовать, все бозоны снова становятся безмассовыми


То есть оно прямо совсем полностью совершенно напрочь отключается? Или действует, но так слабо, что можно его проигнорировать?
12 578565
>>578560
Неясно, вычитывал ли Цио это в учебнике, или допетрил сам, этот ответ я не могу дать.
Но да, примерно за век до отца космонавтики Уильям Мур опубликовал следующее пособие:
https://books.google.ru/books?id=nrVgAAAAcAAJ&printsec=frontcover&hl=ru#v=onepage&q&f=false
Примерно в середине ты найдёшь примерно ту же формулу.

Разница в отношении к Муру и Цио обоснована тем, что Мур писал учебник для баллистических расчётов, а Цио применительно к полёту в космос, и ещё и многоступенчатый вариант.

Отвечая на твой вопрос: возможно он и видел эту формулу, возможно вывел сам, т.к. она была на поверхности, но именно он публиковался с этими формулами как способы расчётов для космических полётов.
13 578567
>>578561

>При энергиях выше нескольких сотен ГэВ механизм Хиггса перестает действовать, все бозоны снова становятся безмассовыми и оба взаимодействия начинают вести себя абсолютно одинаково.


Холд он, мейт. У меня шаблон порвался от такого.
А не наоборот? При ГэВ может фотоны начинают проявлять эффекты массовых частиц? Ведь при высоких энергиях у массовых частиц увеличивается (забыл как называется) масса, ну и продолжительность жизни из-за релятивистской скорости. Эта продолжительность влияет что ли?

>Сложный прикол, короче, на этом поле не одна нобелевка получена.


Посоветуй видео на ангельском, если можешь. Я на русском физику воспринимаю только в формате общения на дваче как рудимент.

>Это так называется большой разрыв между...


Уф, бля, вот я балбес, мог же загуглить сам.
Да, понимаю эти порядки. Класс.
Вот из-за таких вещей как 13 порядков энергии, нейтронная материя, захват пространства во вращение и т.д. я и люблю эту тематику.
14 578577
>>578517 (OP)
а какая эпоха на втором пике после эпохи галактик, та, которая сейчас?
15 578579
>>578577
Эта эпоха обычно обозначается словом "ПОДАРОК".
16 578580
>>578579
а более поэтичное есть что?
17 578581
>>578564

>То есть оно прямо совсем полностью совершенно напрочь отключается? Или действует, но так слабо, что можно его проигнорировать?


Совершенно отключается, при температуре выше критической никакого спонтанного нарушения симметрии не происходит и поле Хиггса с объединенными бозонами никак не взаимодействует.

>А не наоборот? При ГэВ может фотоны начинают проявлять эффекты массовых частиц? Ведь при высоких энергиях у массовых частиц увеличивается (забыл как называется) масса, ну и продолжительность жизни из-за релятивистской скорости. Эта продолжительность влияет что ли?


Нет, не наоборот — механизм Хиггса именно что добавляет массу W- и Z-бозонам. Там совершенно другой принцип, не как у Эйнштейна в ОТО, я когда-то давно пытался разобраться, но сейчас уже нихуя не вспомню.

Посоветовать, к сожалению, тоже ничего не могу, остается только ждать, что придет какой-нибудь нормальный физик и пояснит нам, что почитать на эту тему.
18 578582
>>578581
Вторая часть была сюда → >>578567
19 578583
>>578580
Есть песня Пинки Пай включающая игру слов present/presents https://youtu.be/BOKKisGbkDQ но к астрономии она не совсем актуальная.
Оригинальный вопрос отвечен на втором пике, текущая эра не имеет общепринятого названия, эпоха просто настоящая, то, что происходит сейчас. Звёзды зарождаются и умирают, миллиарды лет проходят.
Вот через триллион наступит эпоха красных карликов.
Дальнейшее будущее довольно тёмное.
20 578584
>>578581

>Совершенно отключается, при температуре выше критической никакого спонтанного нарушения симметрии не происходит и поле Хиггса с объединенными бозонами никак не взаимодействует.


Бля! Ссылку на статью, пожалуйста, занесу в закладку! Или PBS Space Time если был. Я сейчас слегка очень подвыпивший, чтобы с ходу понять суть в достаточной степени, но завтра познакомлюсь поближе.

>Нет, не наоборот — механизм Хиггса именно что добавляет массу W- и Z-бозонам. Там совершенно другой принцип, не как у Эйнштейна в ОТО, я когда-то давно пытался разобраться, но сейчас уже нихуя не вспомню.


Я прекрасно понимаю тебя. Я два раза умудрился разобраться и понять почему сверхсвет нарушает принцип причинности, и мне надо было полтора листа А4 чтобы это донести как мне казалось другому человеку.
Через неделю я мог донести этим листом информацию лишь до себя.
Через месяц я не понимал, как сверхсвет нарушает принцип причинности.
С квантомехом вообще пиздец, я мог формулы смотреть и считать, но понимать я никогда не мог. Оно работало, я осознавал, но я не понимал, и я даже не представлял, что делать, чтобы понять.
Я смог понять и представить четвертое измерение будучи школьником без сопроводительной литературы и математики, блин, я думал, что могу абстрагироваться.
Этот мясной мозг, блин.

Потому и напился, в том числе, извините.
21 578585
>>578581

>Совершенно отключается, при температуре выше критической никакого спонтанного нарушения симметрии не происходит и поле Хиггса с объединенными бозонами никак не взаимодействует.


А как это стыкуется с ОТО? Ведь энергия частицы зависит от системы отсчета, то есть, как я понимаю тут в одной системе поле Хиггса взаимодействует с бозонами, а в другой - нет. Или это из области колдунства квантмеха?
22 578587
>>578584

> сверхсвет нарушает


алькуберка из оп-поста смотрит на тебя как на долбоёба
23 578594
>>578587
В альбукерке нет сверхсвета.
24 578596
>>578594
манёвр терминологией?
или ты опровергаешь существование маскота ттв?
25 578597
>>578596
Ну, можно сказать что и маневр. Но относительно любой ИСО альбукеркнутый корабль движется с досветовой скоростью, и принцип причинности не нарушается. Сверхсвет будет "в итоге", после скукоживаний/раскукоживаний пространство, но это не то, он с СТО не связан.
26 578598
>>578597
ну да, проблема невозможности наблюдения сверхсвета в СТО некрасивая штука
а ведь даже в плане передачи информации вполне легальный сверхсвет
27 578619
А насколько хорошая идея отказываться от аналогового телевещания целиком и полностью? Какие подводные камни есть, почему это может аукнуться?
28 578634
>>578619
Если ты о переходе зомбоящика на DVB-T2, то зона покрытия немного уменьшится. А для увеличения зоны покрытия до прежнего уровня потребуется на порядок больше денег. Пруфов не будет, наверное.
29 578642
Это нормально, что мне башку взрывает то, как корабли переходят на более высокие орбиты? Тип, два раза ускоряется по направлению движения, а в итоге скорость меньше. Пиздец.
30 578654
>>578642
Схуяли она меньше? Не путаешь ли ты орбитальную и угловую скорость?
31 578656
>>578654
Нет, не путает.

>>578642
Ну так а потенциальная энергия больше. Ровно то же самое, как если ты камень бросаешь вверх, то есть разгоняешь его, а он вверху вообще останавливается.
32 578666
>>578642
Это нормально, что тебе башку взрывает. Это не интуитивно, пока не привыкнешь, пока не дойдет.
На деле игра в огурцов вполне доносит это и делает это интуитивным.
Да и ИРЛ ты можешь выйти на суборбиту и в апогее иметь скорость 0.
33 578667
>>578619

>>>578638


Что было в этом посте?
34 578879
Прошу ответить понятным языком как человеку который вообще ноль в космонавтике. Вопрос естественно тупой и максимально детский.
У человечества вообще есть шансы на то что хотя бы в этом тысячелетии бороздить просторы космоса за пределом солнечной системы?
Есть ли хоть какое то топливо на земле которое позволит преодолевать миллиарды световых лет?
Как потенциально будет организовываться колонизация, ну хотя бы того же марса, ну или планеты на которой пригодные для жизни условия?
35 578906
>>578879

>У человечества вообще есть шансы на то что хотя бы в этом тысячелетии бороздить просторы космоса за пределом солнечной системы?


Да, есть шансы на то, чтобы в этом тысячелетии выйти за пределы солнечной системы.

>Есть ли хоть какое то топливо на земле которое позволит преодолевать миллиарды световых лет?


И нет, и да.
При нужных маневрах ты можешь даже на сжатом воздухе улететь на миллиард светолет.
Прочитай лучше про такое понятие, как "характеристическая скорость".

>Как потенциально будет организовываться колонизация, ну хотя бы того же марса, ну или планеты на которой пригодные для жизни условия?


Пока что потенциально план заключается в создании небольшой базы уровня МКС на которой ротация каждый год или полгода, это обсуждалось в космических агентствах.
Планов по терраформированию нет ни у кого вообще даже на бумаге, даже в обсуждениях на серьёзных щщах.
Если маняпланы, то фокусирующая солетта, т.е. гигаструктура, которая будет передавать солнечный свет с большего радиуса, чем марс бы получал сам.
Другой вариант - парниковый эффект.
Либо кометами, либо мохолами.
Это всё опять же гигаструктуры.
Для сравнения - выпустить абсолютно весь ядерный потенциал всех стран земли по полярным шапкам будет меньше миллионной доли нужной энергии для терраформирования.
Сильно меньше миллионной доли.
36 578959
>>578879
О, я тебе объясню. Без этой "философской" дилетантской хуйни что тебе напишут.
К звёздам, но только ближайшим, можно долететь на термоядерном топливе. Для этого нужен термоядерный двигатель-реактор. ITER не подойдёт, нужен реактор в виде трубы с сердцевиной, в которой происходит ядерное горение. Горячие продукты реакции с одного конца трубы вылетают через магнитное сопло, создаётся тяга.
Такие реакторы уже есть (реакторы, без сопла). Это экспериментальные установки типов "Открытая ловушка" и "Обращённая магнитная конфигурация" (FRC). Ты можешь почитать сам.
Они уже работают, но плохо. Но не так чтобы уж совсем плохо, кстати.
Чтобы довести их нужно лет 30-40-50. Собственно, в такой вот перспективе мы и сможем создавать термоядерные двигатели.
Но даже с термоядерными двигателями летать будет очень сложно! Слишком уж велика скорость, не хватает энергии, заключённой в термоядерном топливе.
Пример. Двигатель на самом лучшем химическом топливе (кислород/водород) даёт скорость истечения газов из сопла 4,5 км/с. Чтобы вывести груз на орбиту, нужно набрать 9 км/с. Получается, ракета может разогнаться до скорости только в 2 раза большей, чем скорость газов, вырываюшихся из двигателя. Это почему? Потому что даже такая ракета сможет вывести груза около 5-6% от собственной массы, это очень мало. Так что лететь быстрее можно, но нецелесообразно. Полетишь чуть быстрей, и не сможешь вывести вообще ничего.
Заметь, ракету я взял на самом лучшем топливе.
Теперь к термоядерным ракетам. Самое лучшее топливо здесь это дейтерий/гелий-3. Или гелий-3/гелий-3, пока сложно сказать. Скорость истечения будет около 5-7% от скорости света.
То есть получается, термоядерная ракета разгонится до двух скоростей истечения или до 10% скорости света. Это ОЧЕНЬ ПЛОХО! До самой близкой звезды тебе лететь 40 лет, гигантский срок.
А сколько колонистов доставит ракета? 5% от собственной массы, как мы раньше считали? А вот и нет, 5%, это если бы мы только разгонялись до 10% световой. А нам нужно ещё и затормозить по прилёту. То есть 5% от 5%, это 0,05 * 0,05 = 0,0025. Маса груза в 400 раз меньше массы ракеты, немыслимо! Кого ты хочешь запустить, 10 человек в криокапсулах, 10-тонную посадочную капсулу и 10 тонн груза (это очень мало, кстати)? Ок, около 30 тонн, ииии ракета должна быть в 12 000 тонн.
Ты понимаешь, насколько это хуёво? Во сколько обойдутся 12 тыс. тонн гелия-3, где бы его ни добывали? Эта сумма для 10 человек, пассажиров, будет абсолютно неподъёмна. И лететь 40 лет, анон, почти всю жизнь лететь.
Вот такой вот положняк. Всё это грустно. Но долететь возможно. Уже к концу нашего века.

А на Марсе, в грунте и мерзлоте, могут быть большие запасы газов. Вмороженных, так сказать. В связанной форме. Метан и углекислый газ. Если это действительно так (кстати, вероятность есть), то всю эту замёрзшую грязь потребуется растопить, газ выйдет и давление атмосферы поднимется. Давление это единственная проблема, которую нужно решить, чтобы там жить. Марс не нужно делать теплее, ок? Он и так достаточно тёплый, там на экваторе +20 летом бывает (ненадолго).
Так вот, потребуется очень много энергии. Что-то около "5 тысяч реакторов работают 50 лет", но это всё можно сделать, вполне. 20 лет ты их постепенно строишь, 50 лет они работают и - вуаля, к концу века получаешь пригодный Марс. Кислорода там не будет, конечно. Но ходи себе в кислородной маске и куртке, чем не жизнь? А сейчас для Марса нужен скафандр, если ты его порвёшь, ты умрёшь через минуту. То же самое и на Луне, Марс = Луна = вакуум, вот как всё безрадостно.
Реакторы для растапливания мерзлоты могут быть термоядерные. 5 тысяч 1-Гигаваттных реакторов, где-то так. Работать могут на местном дейтерии, на Марсе много дейтерия, в 5 раз больше в марсианской воде, чем в нашей воде.
36 578959
>>578879
О, я тебе объясню. Без этой "философской" дилетантской хуйни что тебе напишут.
К звёздам, но только ближайшим, можно долететь на термоядерном топливе. Для этого нужен термоядерный двигатель-реактор. ITER не подойдёт, нужен реактор в виде трубы с сердцевиной, в которой происходит ядерное горение. Горячие продукты реакции с одного конца трубы вылетают через магнитное сопло, создаётся тяга.
Такие реакторы уже есть (реакторы, без сопла). Это экспериментальные установки типов "Открытая ловушка" и "Обращённая магнитная конфигурация" (FRC). Ты можешь почитать сам.
Они уже работают, но плохо. Но не так чтобы уж совсем плохо, кстати.
Чтобы довести их нужно лет 30-40-50. Собственно, в такой вот перспективе мы и сможем создавать термоядерные двигатели.
Но даже с термоядерными двигателями летать будет очень сложно! Слишком уж велика скорость, не хватает энергии, заключённой в термоядерном топливе.
Пример. Двигатель на самом лучшем химическом топливе (кислород/водород) даёт скорость истечения газов из сопла 4,5 км/с. Чтобы вывести груз на орбиту, нужно набрать 9 км/с. Получается, ракета может разогнаться до скорости только в 2 раза большей, чем скорость газов, вырываюшихся из двигателя. Это почему? Потому что даже такая ракета сможет вывести груза около 5-6% от собственной массы, это очень мало. Так что лететь быстрее можно, но нецелесообразно. Полетишь чуть быстрей, и не сможешь вывести вообще ничего.
Заметь, ракету я взял на самом лучшем топливе.
Теперь к термоядерным ракетам. Самое лучшее топливо здесь это дейтерий/гелий-3. Или гелий-3/гелий-3, пока сложно сказать. Скорость истечения будет около 5-7% от скорости света.
То есть получается, термоядерная ракета разгонится до двух скоростей истечения или до 10% скорости света. Это ОЧЕНЬ ПЛОХО! До самой близкой звезды тебе лететь 40 лет, гигантский срок.
А сколько колонистов доставит ракета? 5% от собственной массы, как мы раньше считали? А вот и нет, 5%, это если бы мы только разгонялись до 10% световой. А нам нужно ещё и затормозить по прилёту. То есть 5% от 5%, это 0,05 * 0,05 = 0,0025. Маса груза в 400 раз меньше массы ракеты, немыслимо! Кого ты хочешь запустить, 10 человек в криокапсулах, 10-тонную посадочную капсулу и 10 тонн груза (это очень мало, кстати)? Ок, около 30 тонн, ииии ракета должна быть в 12 000 тонн.
Ты понимаешь, насколько это хуёво? Во сколько обойдутся 12 тыс. тонн гелия-3, где бы его ни добывали? Эта сумма для 10 человек, пассажиров, будет абсолютно неподъёмна. И лететь 40 лет, анон, почти всю жизнь лететь.
Вот такой вот положняк. Всё это грустно. Но долететь возможно. Уже к концу нашего века.

А на Марсе, в грунте и мерзлоте, могут быть большие запасы газов. Вмороженных, так сказать. В связанной форме. Метан и углекислый газ. Если это действительно так (кстати, вероятность есть), то всю эту замёрзшую грязь потребуется растопить, газ выйдет и давление атмосферы поднимется. Давление это единственная проблема, которую нужно решить, чтобы там жить. Марс не нужно делать теплее, ок? Он и так достаточно тёплый, там на экваторе +20 летом бывает (ненадолго).
Так вот, потребуется очень много энергии. Что-то около "5 тысяч реакторов работают 50 лет", но это всё можно сделать, вполне. 20 лет ты их постепенно строишь, 50 лет они работают и - вуаля, к концу века получаешь пригодный Марс. Кислорода там не будет, конечно. Но ходи себе в кислородной маске и куртке, чем не жизнь? А сейчас для Марса нужен скафандр, если ты его порвёшь, ты умрёшь через минуту. То же самое и на Луне, Марс = Луна = вакуум, вот как всё безрадостно.
Реакторы для растапливания мерзлоты могут быть термоядерные. 5 тысяч 1-Гигаваттных реакторов, где-то так. Работать могут на местном дейтерии, на Марсе много дейтерия, в 5 раз больше в марсианской воде, чем в нашей воде.
Игорян 37 578962
Ок, будьте вольны ответить плез. Почему в нашей галактике планеты названы красиво по типу Юпитер, Марс, Нептун, Меркурий, а в других галактиках астрономы зовут это так, будто случайные буквы на клавиатуре нажали?
38 578963
>>578879

>У человечества вообще есть шансы на то что хотя бы в этом тысячелетии бороздить просторы космоса за пределом солнечной системы?


Загадывать на срок больше десятка лет - все равно что гадать на кофейной гуще. Больше сотни - получаешь Лондон под метровым слоем конского навоза. За тысячу лет может случиться цивилизация тольтеков на ховербордах и семь ядерных войн. Так что лучше не стоит вскрывать эту тему.
39 578965
>>578962
Другие галактики не видно, они хоть и огромные, но очень далеко. И открыли их поздно, когда телескопы мощные появились. Вот и получается, что это уже были современные времена, 19-20 век. Это в древности древний астроном давал поэтичные названия. А в 19 веке ты, как астроном-учёный, просто берёшь и составляешь каталог: Объект-1, Объект-419 и т. д. Потом каталог называют в твою честь: Каталог Джона Грина, JG. А объекты твои именуют JG0017, JG12847 и т. д.
40 578966
>>578963
Вооо, это и есть чистейшей воды "философская дилетантская хуйня". Такое лучше вообще не писать (если тебе, по-факту, нечего сказать). И уж точно ТАКОЕ не нужно читать и задумываться над этим.
41 578968
>>578962
Планеты в других галактиках пока обнаружены не были, даже в соседних, тех что совсем близко. Не хватает чувствительности инструментов для этого. Есть только косвенные свидетельства и общее понимание что они там есть. Планеты гарантированно найдены только около относительно близких звёзд в нашей галактике.

Названиями небесных тел заведует Международный астрономический союз (IAU). У него есть свои правила. Названия экзопланет даются по названию звёзд, вокруг которых они обращаются, и букв a, b, c и т.д. Звёзды называются по индексам, под которыми они числятся в разных астрономических каталогах. (их с десяток-другой больших и еще куча поменьше).

Тела в Солнечной системе были названы задолго до астрономии как современной науки, а те что были обнаружены современными астрономами - названы потому что они имеют большее значение. Экзопланет же огромная куча, осмысленных названий на всех не хватит, и наблюдать их сложно. (первому прямому наблюдению экзопланеты всего 16 лет). Поэтому они пока числятся по индексам, понадобится - присвоят осмысленное название. У некоторых оно и так есть, например Проксима b (вторая планета ближней к нам звездной системы).
15162698261650.png57 Кб, 600x600
42 578975
>>578966
Дилетантская хуйня - это твои рассуждения о термоядерных двигателях, которые на текущем этапе даже неизвестно можно ли вообще сделать, или упрёшься в фундаментальный лимит материалов или термодинамики. В который упирается любой эффективный тепловой движок, ты даже ни слова про радиаторы, которые требуются такой йобе, но уже готов всё расписать и сколько лет и тонн на это надо.

Поэтому сразу сказать, что ХУЙ ЕГО ЗНАЕТ, будет намного честнее, чем разводить тотальный сайфай с гелием-3.
43 578977
>>578959
Зачем же ты так приложил этого анона с любопытствующим вопросом, который спрашивает у полноценных интеллектуально но не психически здоровых мужчин? Он же сейчас порвётся.
44 578988
>>578959

>вуаля, к концу века получаешь пригодный Марс. Кислорода там не будет, конечно. Но ходи себе в кислородной маске и куртке, чем не жизнь?


А радиацию сплевывай в песочек, отодвигая маску.
45 578991
>>578988
Подожди, сейчас у него будут магнитные Нью-Васюки с лентой по экватору. Хули там, терраформирователю двести верст не крюк.
46 579000
>>578988
Кстати, а созданная толстая атмосфера не будет лучше защищать от радиации?
47 579004
>>578619
Аналоговый сигнал больше подвержен помехам, но намного лучше работает в условиях этих самых помех и плохого приёма (лучше деградирует).
48 579013
>>578968

>первому прямому наблюдению экзопланеты всего 16 лет


Вот это-то всегда и греет. Лет сто назад искусственные каналы на Марсе не могли ни доказать, ни опровергнуть, а теперь ныряют в Юпитер и пырят на квазары. Но я думаю что судьба межзвездных перелетов лежит скорее в духовной плоскости, нас ждет слом парадигмы от которого охуеют вообще все, в 1000 раз круче Эйншетйна. И почему мы их не слышим сразу же всем станет понятно. Токо бы дожить.
49 579029
>>578959
Странный способ считать дельту.
50 579032
>>579004

>лучше деградирует


Прямо как я.
51 579045
>>579000
Да, будет. Давление нужно 1/15 атм. минимум, тогда сможешь в маске дышать чистым кислородом. Это 380г/см2 газового столба (с учётом марсианской гравитации). А на Земле 1000 г/см2, всего в 3 раза больше.
При проектировании космического аппарата хорошей защитой считается 20-30 г/см2 алюминия. Алюминий плохо защищает, чем легче ядра, тем лучше. В атмосфере из СО2 и СН4 все ядра сильно легче Al, а водород это вообще самая эффективная защита.
В общем, 380 г/см2 заведомо заебатая защита. Я не считал, придётся поверить на слово. Но это треть земной атмосферы по массе, это где-то высота Эвереста. На высоте Эвереста космическая радиация абсолютно безопасна.

Радиаторы охлаждения >>578975 я прикидывал, более менее норм: температура 2500 градусов, керамика, циркулирует жидкое олово, например. Радиаторы не тяжёлые. Вот удельную мощность реактора сложнее оценить. А ты, с петухом на аве, нахуй иди. Если тебе так понятнее, лол)
52 579046
>>578988
С хуя ли там будет радиация при плотной атмосфере?
53 579047
>>579029
Для лучшего понимания.
54 579048
>>579047
Но я и так понял, что ты дегенерат.
55 579052
>>578991
Ля, у него нет магнитных Васюков, ему похуй просто >> >>579046
56 579062
>>579052
Ты дурак? Атмосфера защищает от радиации намного лучше магнитного поля, на магнитное поле вообще поебать, если есть атмосфера. Поле задерживает только заряженные частицы, которые и так в веществе быстро поглощаются, а незаряженные задерживает только атмосфера, что здесь, что на Марсе, что на Венере.
57 579063
>>579045

>На высоте Эвереста космическая радиация абсолютно безопасна.


Ну это тоже не совсем верно, там радиация заметно выше, чем на поверхности Земли.
58 579065
>>579062
Более того, северное сияние происходит на совсем большой высоте, километров 50. Представь, какая ничтожно тонкая атмосфера там? Но даже её хватает чтобы остановить эти частицы. А вот Галактические космические лучи задерживаются хуже. Для этого-то, собственно, такая толстая атмосфера нам и нужна.
Магнитное поле что есть, что нет, на уровень радиации у поверхности не повлияет никак.
59 579067
>>578879

> за пределом солнечной


шансы хорошие даже примерно чувствую, если не долететь до ближайшей звезды от Солнца, то хотя бы вылететь за пределы нашей

> будет организовываться


вначале будет небольшой посёлок из привезённых банок возле марсианской дырки, в которую довольно быстро переселятся
ну или же не будут заморачиваться с дырками, а просто выкопают землянки марсианки с герметизацией для колонистов
впоследствии будут проводится работы по углублению и расширению жилого и рабочего фондов и гео марсоразведка для постройки следущих поселений возле залежей ископаемых, необходимых для самообеспечения
алсо будут завозить специфический адсорбент для выведения из почвы перхлоратов, ну или опять же не будут заморачиваться и просто привезут земли
алсо если и будет ротация - то кратная окнам, по 3 года примерно
ещё каких-то прохладных расказать? на самом-то деле много организационных вопросов
60 579069
>>579063
А я писал что не выше?
61 579080
>>578879
Итак, все ответы выше так или иначе говно.

>У человечества вообще есть шансы на то что хотя бы в этом тысячелетии бороздить просторы космоса за пределом солнечной системы?


Да.

>Есть ли хоть какое то топливо на земле которое позволит преодолевать миллиарды световых лет?


Дело тут не в волшебном топливе, а в том, что на разгон нужно тратить энергию, много энергии. И чем выше скорость, тем больше возрастают затраты, причём так, что по мере приближения к скорости света затраты энергии на разгон устремляются в бесконечность, буквально, то есть нужно было бы затратить бесконечно много энергии, чтобы достичь скорости света (это если у тебя есть какая-либо масса, а если нет, то ты и так летаешь со скоростью света). Со скоростью света миллиарды световых лет ты будешь преодолевать за миллиарды лет, и быстрее никак. Правда, есть нюанс. Миллиарды лет будут проходить для стороннего наблюдателя, а для тебя не пройдёт и секунды. С околосветовой скоростью можно летать в другие галактики за сроки, ограниченные только скоростью разгона и торможения, и если разгоняться и тормозить с ускорением g, то будешь тратить на весь путь порядка 10 лет (с точки зрения экипажа корабля, в остальном же мире могут пройти сотни, тысячи, миллиарды лет). Но, конечно, не стоит думать, что чтобы бороздить галактики нужно обязательно укладываться в сроки, которые мы сейчас считаем сроками человеческой жизни. За тысячу лет могут научиться продлевать жизнь, переносить сознание в компьютер, да мало ли что ещё. В крайнем случае можно построить гигантский корабль, на котором сменится много поколений, пока он будет лететь. А если тебе не важно, сколько времени лететь, то можно и с современными двигателями летать к звёздам.

Да, вот ещё, разгоняться до межзвёздных скоростей можно с помощью гравитационных манёвров, у планет-гигантов, у близких звёзд, у чёрных дыр и т.п.
62 579082
>>579069
Ты написал, что "абсолютно безопасна". Однако у пилотов, которые на примерно тех же высотах летают, есть допустимые нормы по облучению.
63 579091
>>579082
Пхех, а давай я тебе скажу что и на уровне моря есть космическая радиация? Не, ну есть же! Давай тогда и её будем считать опасной? Не, не будем, "люди испокон веков так живут" ? Ну вот они и в горах испокон веков живут и раком не болеют.
А известно тебе что фоновая радиация в разных местах разная, и кое где даже выше чем на Эвересте. И ничего, всё так же "живут, не болеют". Вот что я и подразумеваю под определением "абсолютно безопасна". Можешь со мной не соглашаться, дело твоё. Можешь также посмотреть, насколько сильно отличаются нормы у работников атомной промышленности и у обычных людей. Нормы, знаешь ли, понятие размытое.
64 579096
>>579091

> раком не болеют


нахуй иди, сектант
65 579101
>>579080

>разгоняться до межзвёздных скоростей можно с помощью гравитационных манёвров, у планет-гигантов


Совершеннейшее враньё!

>у близких звёзд


враньё, 2000 км/с (в лучшем случае) это не межзвёздная скорость

>у чёрных дыр


ещё один пустоголовый дилетант, meeehh
Алё, йоба! Это я! Выхожу на связь из Солнечной системы. Здесь нет чёрных дыр, повторяю, в радиусе 1000 световых лет нет чёрных дыр! Йоба, алё, приём, приёёём!
66 579102
>>579096
иди нахуй, цепляющийся к словам мудачок
67 579104
>>579101

> 2000 км/с


> не межзвёздная


да, это уже межгалактическая так-то
68 579119
>>579104
уже ответил постом выше
69 579125
>>579101

>Совершеннейшее враньё!


Ты дебил? Как по-твоему Вояджеры скорость улетания из СС набрали?

>враньё, 2000 км/с (в лучшем случае) это не межзвёздная скорость


Да, как уже тебе ответили, это межгалактическая.

Пиздец, какой же ты дебил, просто поражаюсь.
1591125851923.jpg187 Кб, 604x604
70 579135
Можно ли на двигателе Альбукерке вылететь из черной дыры?
71 579155
>>579135
Можно, если извинишься перед обитателями за 200 лет рабства.
72 579169
>>579000
Атмосферу сдует нахуй солнечным ветром, её собственно и нет на марсе из-за слабого магнитного поля.
73 579172
>>579169
Сдувание атмосферы займет сотни миллионов лет, на наш век хватит.
74 579189
>>579155
бля, а какие сверхлюди не прошли через феодализм и сопутствующее ему рабство?
75 579190
>>579125
Да нет, чел, это ты удивляешь. Ты про "корабли поколений" начитался чтоли? Увы, огорчу тебя, это нежизнеспособная концепция. Почему? Потому что Я так скозал. Ну и потому что лететь up to 100000 лет, тут как бы и обсуждать нечего, лол.
А какую скорость НА САМОМ ДЕЛЕ называть "межзвёздной"? 10% скорости света, например. 30 000 км/с, например. А не 3000 и не 2000. Такое годится только по своей системе летать, к Нептуну.
76 579219
>>579190
Мань, уймись уже.

>А какую скорость НА САМОМ ДЕЛЕ называть "межзвёздной"?


Ту, на которой можно летать между звёздами. То есть скорость на которой можно улететь от ближайшей звезды и выйти на орбиту вокруг центра Галактики. Всё, остальное - это твои кукареки, не несущие никакого смысла. Чем скорость в 10% с принципиально отличается от скорости 5% с? Да ничем.

>Потому что Я так скозал.


Ну и всё значит, иди нахуй.
1200px-Casimirplates.svg.png150 Кб, 1200x1229
77 579280
На сколько реально собрать "вечный" двигатель на эффекте казимира? Энергия вроде не из ниоткуда берется, а из флуктуаций вакуума. Я нихуя не понял протеворечия с термодинамикой тут есть или нет.
78 579281
>>579280
Как этот эффект можно использовать в двигателе?
79 579284
>>579280
>>579281
Нахуй.
80 579290
>>579281
При определенных конфигурациях эта ебала не притягивается друг к другу, а вращается. Если их дохуя собрать то наверное можно генератор вращать.
81 579292
>>579290
Не семени, говно.
82 579294
>>579290
Вращается? Это как сделать?
83 579295
>>579294
https://www.nature.com/articles/s41586-018-0777-8
Не ебу как, буржуи хотят 9$ за свои секреты, но там что-то связанное с оптически анизотропными кристалами.
изображение.png252 Кб, 420x263
84 579296
1317456087608.jpg121 Кб, 505x600
85 579301
>>579280
Это же концепт насоса для надувания пузыря Альбукерке. Вакуум, процеженный между двумя зеркалами, и не содержащий всяких лишних флуктуаций, можно распылять перед кораблём. Облако вакуума локально повысит скорость света и позволит сильнее разогнаться.
86 579302
>>578959

>А на Марсе, в грунте и мерзлоте, могут быть большие запасы газов. Вмороженных, так сказать. В связанной форме. Метан и углекислый газ. Если это действительно так (кстати, вероятность есть), то всю эту замёрзшую грязь потребуется растопить, газ выйдет и давление атмосферы поднимется


Поднимется, причем в разы. Но для человека это все равно будет вакуум. Если испарить весь со2, н2о и остальные газы давления будет несколько процентов от земного.
87 579306
>>579169
Зелёный кот считал, нет. Магнитное поле переоценено. Сдуло а на самом деле не сдуло, а улетело само из-за малой массы планеты, совокупно с высокой температурой атмосферы.
88 579313
>>579306
Разве из-за малой массы? У Титана тогда почему не улетело?
89 579333
>>579313
Потому что на Титане холодно. Скорость теплового движения молекул атмосферы Титана при 100К получается меньше второй космической.
90 579336
>>579333
А на Венере тогда почему не улетело? Там температура еще выше чем на Марсе а масса не особо больше.
91 579344
>>579336

>масса не особо больше.


примерно в 10 раз.
92 579346
>>579284
Ты чё порвался-то?
93 579370
>>578517 (OP)
Раньше не интересовался астрологией, вот сейчас почитал немогу понять. Почему газовые шарики - планеты, а плутон нет? А Звезды - планеты? А если я перну в космосе это будет планетой?
94 579385
>>579336

>А на Венере тогда почему не улетело?


Вот именно. Магнитного поля то там нет.
95 579389
>>579370

>Почему газовые шарики - планеты, а плутон нет?


По современному определению планеты, которое было утверждено в 2006 году. Плутон не является гравитационно доминирующим телом в окрестности своей орбиты, то есть не расчистил орбиту от других планетезималей.

>А Звезды - планеты?


Нет, потому что достаточно массивны, чтобы в них поддерживалась реакция термоядерного синтеза.

>А если я перну в космосе это будет планетой?


Нет.
96 579392
>>579302
Ты не знаешь о чём говоришь, у тебя нет цифр. Ты простой трепач. И в этом разница между нами, ведь я знаю о чём говорю и оперирую конкретными цифрами. Без обид.
97 579394
>>579389

>По современному определению планеты, которое было утверждено в 2006 году. Плутон не является гравитационно доминирующим телом в окрестности своей орбиты, то есть не расчистил орбиту от других планетезималей.



Т.е. вопрос в наличии достаточно сильной гравитации и отсутствии термоядерной реакции?
98 579396
>>579394

>в наличии достаточно сильной гравитации


Относительно говна вокруг, да.

>и отсутствии термоядерной реакции?


Да. Есть ещё коричневые карлики, это очень массивные газовые гиганты или недозвёзды, в которых термоядерные реакции могут происходить непродолжительное время после образования, но быстро гаснут. Их обычно не причисляют к звёздам, то есть это планетарные объекты.
99 579397
>>579392
Шиз, в твоём высере про Марс не было ни одной цифры.
100 579399
>>579396

>Относительно говна вокруг, да.


Еще и относительно себя, чтобы самоскукожиться в кругляш.
изображение.png329 Кб, 512x400
101 579400
>>579396
Спасибо, не находил такого понятного объяснения
102 579401
>>579397
Читай лучше, или иди нахуй
103 579434
>>579045

>Алюминий плохо защищает, чем легче ядра, тем лучше. В атмосфере из СО2 и СН4 все ядра сильно легче Al, а водород это вообще самая эффективная защита.


Щито, наоборот надо плотнее.
104 579437
>>579189
Эскимосы.
105 579441
Есть ядерные реакции распада, когда тяжелые ядра делятся, цепные реакции такого рода в ядерных реакторах используют.
Есть ядерные реакции распада спонтанные, такие реакции в РИТЭГах используют.
Есть ядерные реакции синтеза, когда из легких ядер получаются потяжелее, такие реакции используют в Солнце, бонбах и собираются запилить реакторы.
А есть ли еще виды ядерных реакций? Например, распад легких ядер которые нестабильны? Реакции с энергиями порядка килоэлектронвольт, а не мегаэлектронвольт?
106 579443
>>579401

>высер двачного червя-пидора


>Читай


А ты шутник.
1591195010733.png412 Кб, 1080x2280
107 579461
>>579441
у тебя очень всратая классификация
есть реакции как синтеза и распада, они могут иметь как разный энергетический баланс, так и различные продукты на входе и выходе

да, лёгкие ядра тоже имеют нестабильные изотопы и распадаются

реакции с энергиями в килоэлектрон вольты есть, собсна пикрел
108 579466
>>579443
Ну тогда не читай и иди нахуй.
109 579467
>>579370

> астрологией


Ебло ослиное
110 579468
>>579467
Астрология - это наука о звездах, учение.
Астрономия - это какой-то закон, это не наука.
Называйте вещи своими именами.
Пидорасов с их картами таро просто расстреляйте и забудьте о их существовании.
111 579480
>>579468
А есть наука Троллесофия?
112 579482
>>579480
Сперва Бугуртоведение, а потом уже по специальностям, пранкология, прикладное ожирение, ядерная бугуртистика, и т.д.
113 579493
>>579482

>атомная бугуртистика

атомный бугурт.jpg24 Кб, 314x292
114 579499
>>579493
А, да, точно.
115 579504
>>578959
>>578966
>>579169
>>579045
>>579046
>>579062
>>579065
>>579080
>>579091
>>579392
>>579401
Вы попутали, это тред для тупых вопросов, а не ответов.
116 579506
Сколько энергии надо затрачивать чтобы удерживать на магнитном подвесе 1 (один) килограмм заряженной антиматерии?
Я чето считаю - 1кг это 10 ньютонов силы, значит надо 10 ватт чтобы удерживать.
10 ватт это как-то маловато, пусть с кпд 10% будет 100 ватт электричества, это же мелочь.
Батарея держит порядка мегаджоуля в килограмме, т.е. килограмм батарей будет удерживать килограмм антивещества 10000 секунд или почти три часа.
Мелочь какая-то.

Где я обосрался?
117 579513
>>579506
Энергия выражается в джоулях.

> Сколько энергии надо затрачивать чтобы удерживать на магнитном подвесе 1 (один) килограмм заряженной материи?


Ноль джоулей, работа не совершается.
118 579514
>>579513
Совершается, постоянно надо прилагать силу равную весу объекта.
119 579523
>>579514
Хорошо: сколько энергии затрачивает стол на поддержание кирпича, лежащего на нём? Сколько энергии затрачивает пружина?
120 579525
>>579523
Нисколько.
121 579527
>>579525
Вот и магнит столько же, та же самая система.
триумф зеленого толстяка.jpg114 Кб, 800x600
122 579529
>>579527
Слишком толсто, ты кого наебать пытаешься?
123 579530
>>579523
бля, только казуистики из школьной программы не хватало

если ты реально тупой и не тралишь - то кирпич нихуя не лежит, а в каждый момент падает и на его поднятие нужно тратить энергию

алсо тут скорее вопрос на самом деле давно не вопрос как всё это говно, желающее разлететься собрать в кучу
124 579531
>>579530

>кирпич нихуя не лежит, а в каждый момент падает и на его поднятие нужно тратить энергию


Так сколько энергии тратится, и откуда она берётся? Кирпич массой 1кг, допустим.

>тут скорее вопрос на самом деле давно не вопрос как всё это говно, желающее разлететься собрать в кучу


А это уже другой вопрос, речь про "магнитный подвес"
125 579532
>>579531
Энергия не тратится, толстячок, т.к. сила с которой кирпич давит на стол компенсируется силой с которой стол давит на кирпич, ака силой реакции опоры.
В случае с магнитным подвесом сила притяжения компенсируется электромагнитом, который на поддержание магнитного поля затрачивает энергию.
126 579535
>>579532
Подожди, так тратится >>579530 или не тратится >>579532 ?

>сила с которой кирпич давит на стол компенсируется силой с которой стол давит на кирпич, ака силой реакции опоры


С магнитом то же самое. Более того, даже фундаментальные силы те же.

>электромагнитом


А это откуда ты его взял? В задаче ничего такого не было.

>который на поддержание магнитного поля затрачивает энергию


А куда эта энергия уходит, во что преобразуется? А сверхпроводящий тратит?
127 579542
Щас тут ещё предложат выйти на орбиту в магнитном поле, как в одном из прошлых тредов, скриньте.
128 579543
>>579535

>А это откуда ты его взял? В задаче ничего такого не было.


Я его и задумал, как ты выключишь магнитное поле постоянного магнита, наркоман? Магнитный подвес на постоянных магнитах не бывает, любая погрешность и обект улетит.
129 579544
>>579543
заряженная частица хоть как улетит из постоянного поля, безо всяких погрешностей
130 579546
>>579542
гаус-ганом? или что-то из омска?
131 579564
>>579543
Пиздец, вот так отвечаешь людям на поставленный тупой вопрос, а тебя обзывают троллем, наркоманом, постят картиночки и пытаются спорить с реально школьной физикой.
Цитирую вопрос ещё раз:

> Сколько энергии надо затрачивать чтобы удерживать на магнитном подвесе 1 (один) килограмм заряженной антиматерии?


Корректный ответ:

>Ноль джоулей, работа не совершается.


А потом оказывается, что имели в виду не волгу, а сто рублей, не в лотерею, а в карты... Нормально поставленный вопрос - половина ответа, задумайся об этом.

Если же включить телепатические способности и ответить на вопрос, который ты ВОЗМОЖНО хотел задать (но не задал):
- если ты имел в виду сверхпроводящий электромагнит сложной конфигурации, удерживающий в вакууме под ненулевым давлением плазму из антиматерии, то для расчета его энергопотребления недостаточно данных, надо знать как минимум его конфигурацию, характеристики плазмы, и всё остальное.
132 579581
>>579564
а, лол, вопрос начался в пасекс-треде и анон не перенёс все условия, а я не учел что кто-то может быть не вкурсе
прости что подозревал что ты тралишь тупостью
133 579634
>>579564
Мимо другой анон: я так понимаю, что магнитное поле никакой работы не производит, и никакой энергии не тратит. Но для поддержания этого поля может тратиться энергия, в зависимости от того, какой магнит (если электромагнит, то по проводам ток гонять, а если постоянный - делать ничего не надо). Так?
134 579704
Как огурцы приучаются спать в зеро-г? На тренажере же больше полутора минут не полетать, так что в космосе они оказываются впервые в постоянном свободном падении, и хоть привычка не обсираться и не паниковать есть, но это же физиологически пиздец неверное состояние, как можно заснуть при этом?
Колёса жрут? Или за несколько часов моск перестает слушать вестибулярку и может заснуть сам?
135 579705
Пиздец вы тут ученые
136 579706
>>579634
совсем не так, в этот момент магнитное поле землюшки изменяет траекторию движения заряженных частиц, летящих от солнца
любое поле является эквивалентом материи-энергии и вместе с произведённой работой изменяет свою конфигурацию
постоянные магниты дают поле, которое самоподдерживает конфигурацию доменов в магните, изменения поля влияют на расположение доменов, так что работа выполняемая полем снижает уморядоченность системы доменов, тем самым изменяя её этропию
137 579708
>>579706
Вернись в обо/сцай, дебил мелкобуквенный.
138 579710
>>579708
нет, ты
139 579714
>>579710
То есть возражений нет.
Точно олигофрен из обо/сцая, хаха.
140 579798
>>578667
мочератор рассмотрел политику в моем примере с переходом с аналового на цифровое ТВ в сложное время в одной стране и последствиях к чему это привело.
За подробностями к тому кто снёс мой пост и меня забанил.
141 579847
>>578517 (OP)
Я пральна понимаю, что МКС пол-года летает ночью над северным полушарием, а днём - над южным, а пол-года наоборот?
142 579866
Я не совсем понимаю, почему пишут что якобы раньше люди как мухи умирали от любой раны пока не изобрели интибиотики. Но я уже тонну раз ранился, и всё ок. Поясните.
А животные? Я слышал что у крокодилов есть какие-то йоба антитела, которые от любых бактерий защищают. А если какой-нибудь голубь поранится - он труп от инфекции или как?
Поясните пожалуйста.
143 579876
>>579866

>я уже тонну раз ранился, и всё ок


Потому, что ты не работаешь в поле или свинарнике, где легко инфицировать рану. И даже без антибиотиков наверняка рану промываешь, возможно дезинфицируешь, и держишь в относительной чистоте. А еще ты кушоешь получше какого-нибудь колхозника семнадцатого века, что напрямую влияет на иммунитет. Ну и просто статистика: даже если ты "тонну раз ранился", уверен, что эта "тонна" не идет ни в какое сравнение с деревенским мужиком, который пашет, режет и потрошит животных, рубит дрова, строит избы и все такое.
144 579882
>>579876
А животные? Вот голубь поранил ногу, он всё?
145 579885
>>579882
Как повезет. Скорее всего, нет. Животные получают раны относительно часты, и в итоге одна из них может стать смертельной. Но не каждая же.
146 579891
>>579885
Окей, я просто голубя поймал который запутался в сетке, освободил его, но он сеткой порезал ногу немного. Вот я и думаю. У меня не было мыла или дезинфекциона, и я его так отпустил, и думаю надо было лучше домой сбегать или так норм. Как думаешь?
147 579990
>>579847
Нет.
148 579991
>>579891
Еслы ты щупал голубя руками, то конечно надо мыло/антисептик, на голубях куча говна, можешь заболеть.
На самого голубя похую, простой порез в подавляющем большинстве случаев ничем не грозит, будет так и летать, пока не сломает шею врезавшись на полном ходу в застекленную остановку.
149 580012
Нашли птиц, которые могут в невесомость, или всем пизда приходит и они барахтаются как кошки?
150 580013
>>580012
Еще небось и срут во все стороны.
151 580014
>>580013
Это они и на планете делают.
152 580015
>>580014
На планете не критично.
153 580016
>>580013
>>580015
Тащемта решение есть.
Уверен, многие не догадывались.
154 580024
>>580016
Вот бы на всех голубей в городе такую хрень надеть.
155 580049
>>580024
Не проще их потравить?
156 580057
Такое чувство что Вебб телескоп никогда не запустят, а когда запустят ракета порвётся.
157 580089
>>580057
Кстати. Они же производство Ариан-5 остановят, так? На чём он будет лететь?
image.png8 Мб, 2880x1917
158 580101
>>580057
Ты недооцениваешь, насколько он ЙОБА.

>>580089
Арианки до 2022 летают пока еще.
159 580102
Почему не передана общественности запись с камер Прогресса, столкнувшегося с МИРом? Почему такие вещи пылятся в архивах, почему никто не снимает док фильмов с ними? Запись-то есть.
160 580105
>>580102
Потому, что у Роскосмоса другая политика гласности, не такая как у НАСА.
161 580108
Поясните за йеллоустоунский вулкан, пишут что он якобы может уничтожить сша. Я почитал вики, а там только то что сша немного профинансировали наса, которые сказали, что ну да может. И?
162 580109
>>580105
Чем она лучше? Почему выбрана такая модель гласности?
163 580110
>>580109
Ничем не лучше.
Выбрана по наследству с СССР.
164 580111
>>580108

>И?


Что и? Пока никаких возможностей "разрядить" такую йобу не просматривается.
165 580112
>>580108

>И?


Хороший вопрос.
Ответ: "да".
166 580113
>>580101
А для каких целей вообще создаётся уэбб? Просто заменить хаббл? Я вообще слышал что хабблу и применений особо нет, делают просто снимки по топовым работам с универов и всё.

>>580112
>>580111
Т.е. угроза уничтожения континента это нормельно?
167 580116
>>580113

>А для каких целей вообще создаётся уэбб?


Телескопить космические тела.

>Просто заменить хаббл?


Не просто. Он в ИК работает и с большей разрешающей. Он не заменяет какой-то телескоп, он добавляет новый взгляд.

>Я вообще слышал что хабблу и применений особо нет, делают просто снимки по топовым работам с универов и всё.


Хаббл постоянно занят, постоянно в работе.

>Т.е. угроза уничтожения континента это нормельно?


Да, это нормально. Хуево, и надо что-то по-хорошему придумать, но это нормально.
168 580126
>>580101

>Арианки до 2022 летают пока еще.


Ну, так мой вопрос в силе.
169 580129
>>580126
Я думал, что ты мог увидеть ответ в моем посте, если читал между строк.

>>580089

> На чём он будет лететь?


На Ариан-5
170 580139
Поясните, в районах северных сияний повышенный уровень радиационного фона или нет? Там же ускоренные частицы, накопленные магнитосферой, влетают в атмосферу, если я правильно понимаю?
171 580140
>>580139

>Поясните, в районах северных сияний повышенный уровень радиационного фона или нет?


Нет.

>Там же ускоренные частицы, накопленные магнитосферой, влетают в атмосферу, если я правильно понимаю?


Примерно правильно. Они светятся в верхних слоях, не долетая десятки километров до поверхности.
У тебя будет выше фон в горах, чем на приполярных областях.
Ну и в Антарктиде, т.к. озоновая дыра и большая высота.
172 580141
>>580140

> Они светятся в верхних слоях, не долетая десятки километров до поверхности.


Ну дык, когда они взаимодействуют с атомами атмосферы, они должны вызывать потоки вторичных частиц. Кроме того должны излучать рентгеновское излучение при торможении об атмосферу.

>У тебя будет выше фон в горах


Это само собой
173 580144
>>580141

>Ну дык, когда они взаимодействуют с атомами атмосферы, они должны вызывать потоки вторичных частиц. Кроме того должны излучать рентгеновское излучение при торможении об атмосферу.


Дельта- и эпсилон-излучения не активнее беты, так что далеко они не добираются тоже. В случае с рентгеном - у заряженных частиц энергии порядка килоэлектронвольтов, большая часть излучения прекрасно гасится атмосферой.
Авроры не вызывают широких атмосферных ливней, энергии не те.
174 580145
альфа лучи это ядра гелия
бета лучи это электроны
гамма лучи это нейтроны
а какие лучи тогда протоны?
175 580146
>>580145
Гамма - это фотоны. Для протонов нет названия.
176 580161
Всё труднее и труднее открывать и делать что-то. Если лет 200 назад потребовался 1 человек, то сейчас нужны тысячи чтобы исследовать и открывать всякое. Скоро наступит предел, так?
177 580171
>>580161
Потому что развиваться нужно не вверх, а вширь только капитализм этому не способствует
178 580173
>>580171
А чем плох капитализм и какие варианты лучше? Часто встречаю что во всем винят некий капитализм просто.
179 580175
>>580173

>А чем плох капитализм


Тем, что не позволяет развиваться.

>и какие варианты лучше?


Коммунизм, очевидно же.
180 580176
>>580173

>А чем плох капитализм


Тем, что способствует накоплению бОльшей части ресурсов у отдельных людей. А один человек по-определению не может правильно распределить ресурсы для оптимизации развития человечества. Не говоря уже о тупых и объективно бесполезных растратах ресурсов на роскошь.

>и какие варианты лучше?


Я бы сказал коммунизм, но меня закидают говном. Человечество не готово к коммунизму сейчас. Так что капитализм в сочетании с сильной социалкой, прогрессивными налогами и тд - это лучшее решение на данный момент.
181 580180
>>580176

>оптимизации развития человечества


А как определить что значит развитие человечества?

>тупых и объективно бесполезных растратах ресурсов на роскошь


Но ведь объекты роскоши создают тонны рабочих мест, так что ресурсы опят ьвозвращаются.

>>580175
>>580176
Я в этом не разбираюсь, но не однократно читал что коммунизм невозможен в принципе и люди, которые бы могли в теории сделать коммунизм - он им был не выгоден вовсе. Плюс коммунизм означает что всё будет пренадлежать еще меньшей группе людей.
182 580183
>>580180
Пока люди управляют, да, не получится.
Надо чтобы управлял непредвзятый ИИ, тоталитарная диктатура бесчувственного неподкупного робота. Тогда можно честно жить.
183 580201
>>580183

>Тогда можно честно жить.


Как бы он не начал с выпиливания всего человечества в рамках оптимизации.
184 580203
>>580201
Всего не надо, именно поэтому в робота надо четыре закона робототехники впилить (без хитровыебанных способов их интерпретировать в свою пользу типа "лчшее будущее для людей это их смерть, это уменьшит их страдания").
185 580205
>>580144
что за дельты и эпсилоны? куда подевалась православная гамма?
186 580208
Как ирл космические ракеты летают без аэродинамических поверхностей? Когда пытаюсь такое запилить в ksp, оно падает и разваливается при малейшей попытке наклонить ракету.
187 580209
>>580205
А что гамме в авроре делать? Там энергии не те.
188 580211
>>580208
Кардан либо верньеры стабилизируют ракету.
189 580212
>>580180

>Но ведь объекты роскоши создают тонны рабочих мест, так что ресурсы опят ьвозвращаются.


Нет, не возвращаются. Смысл экономики - это выхлоп. Деньги крутятся-труд мутится. Бульон - это результат трудов. Построенная ракета, робот, процессор. А при изготовлении роскоши выхлопа нет.
190 580213
191 580218
>>580213
Высокий авторитет контроля с помощью вектора тяги, говорю, че тут непонятного?
Без него все ракеты бы как Протон бороздили казахские степи.
1256737229456.png321 Кб, 1024x768
192 580222
Тупые и ебанутые вопросы.
Если взять все ядерное оружие на планете (ну, пусть 10000 гигатонн в трот. эквиваленте), сложить в кучу и подорвать на Луне (на поверхности), достаточно ли этого будет для заметного изменения ее орбиты? А именно: подорвать так, чтобы она потеряла орбитальную скорость и понизила орбиту в достаточной мере для начала пиздеца на Земле (наводнения, извержения, землетрясения и прочее, от усилившегося влияния лунной гравитации).
193 580226
>>580222

>Если взять все ядерное оружие на планете (ну, пусть 10000 гигатонн в трот. эквиваленте)


Вообще всего бомб на где-то 3гигатонны по прикидкам, но пусть будет 10 тератонн, почему бы и нет.

>сложить в кучу и подорвать на Луне (на поверхности)


Будет пшик, надо либо сделать одну мегабомбу, либо сделать ебический синхронизатор для всех бомб. Срабатывание одной бомбы на долю секунды раньше других выведет другие бомбы из строя и просто разбросает уран.

>достаточно ли этого будет для заметного изменения ее орбиты?


>А именно: подорвать так, чтобы она потеряла орбитальную скорость и понизила орбиту в достаточной мере для начала пиздеца на Земле (наводнения, извержения, землетрясения и прочее, от усилившегося влияния лунной гравитации).


Даже не близко.
4.2×1015 JEnergy released by explosion of 1 megaton of TNT
10 000 000 megatons = 4.2x1022J
Если представить, что вся эта энергия сфокусирована так, чтобы именно изменить скорость, а не рассеивается в стороны, то эта энергия изменит скорость луны ( массой 7.342×1022 kg ) на целых
Е=1/2 mv2; v = √(2E/m) = √(8.4J/7.32kg) ≈ 1.07 m/s

Один метр в секунду это примерно одна десятая процента средней орбитальной скорости луны. Ты едва заметно изменишь эксцентриситет орбиты и создашь новый кратер. Алсо, эджекта с большой вероятностью может выпасть на землю, поломав несколько машин и выбив стекла.
194 580232
>>580226
Как всегда, мощу ядерки преувеличивают. Вот даже если все-все взять с огромным запасом (увеличим количество в тыщи раз, пусть все взорвется ровненько вместе, пусть абсолютно вся энергия пойдет на изменение орбитальной скорости), получается едва видимый пук, как расписали.
195 580235
Как далеко от солнца надо переместить Венеру, чтобы температура на ней примерно была равна земной? Как быстро она остынет, если поместить её в межзвездное пространство?
196 580236

> шапка


> starcolors.png



Вообще-то, цвет звезды с поверхностной температурой 5800 K (жёлтый карлик) — белый (если смотреть из космоса, а не из-под толстой атмосферы). В этом может убедиться каждый, взглянув на снег или на облака.
image.png57 Кб, 769x504
197 580237
>>580236
Желтизна слонца компенсируется синевой неба. Именно так и придуманы белые светодиоды.
198 580239
Конечно, но синева неба тоже от Солнца. Об это и речь. А при взгляде из космоса Солнце белое, так как атмосферное рассеяние не расщепляет его спектр спектр.
199 580240
>>580218
Дык в кспе оно тоже всё есть. Причём, кмк, в нереалистичных пределах крутится. Однако, ракета перерворачивается.
200 580241
>>580240
Центр массы как можно выше ебашь, ничего не будет переворачиваться. Можно для этого задать очередность использования топливных баков, кстати.
201 580243
>>580240
В огурцаче аэродинамика ебанутая, у тебя нос создает больше драга, чем хвост, что и приводит к опрокидосу.
202 580248
>>580240

>в нереалистичных пределах крутится



Вполне реалистичных, ИРЛ двигатели примерно на 5-10 градусов и отклоняются.
203 580250
>>580248
Ты забыл про ССМЕ
204 580251
>>580248
Зато там есть читерские reaction wheels, которые могут бесконечно крутить крафт куда угодно.
Скоростьнебесноготела.gif1,2 Мб, 400x400
205 580316
>>578642
Если бы не твой вопрос ИТТ, я бы над этой хуйнёй и не задумался, так бы и помер тупым.
250internets.jpg389 Кб, 1000x500
206 580395
>>580226
Спасибо, анон, не ожидал что кто-то так подробно с расчетами пояснит.
Мой мир не будет прежним, никогда бы не подумал, что мощь всей ядерки - это настолько ничтожный пук, даже в масштабах системы Земля-Луна.
изображение.png58 Кб, 590x198
207 580421
поясните за батут
208 580430
>>580421
Он заработал.
209 580434
>>580421
Изделие №26 "Батут" - это кодовое обозначение корабля Crew Dragon, разработанного компанией SpaceX, используемое в Роскосмосе. Данным твитом госкорпорация даёт понять, что по результатам их собственного исследования лишь это изделие сможет доставить американских астронавтов на американской ракете с американской земли на МКС, а изделие №27 "Старлайнер" - не сможет ни через пять лет, ни через шесть.
210 580451
>>578517 (OP)
В чем смысл жизни и всего?
211 580453
>>580451
В единении с Богом.
212 580466
>>580453
Как его достичь?
213 580483
>>580453

>с Богом.


>2ch.hk/spc/


Приехали.
214 580488
>>580451
В пицце с пепперони и халапеньо, это же очевидно.
215 580493
>>580488
И обязательно сладкой кукурузой чтобы посыпали!
216 580494
>>580466
Вкушанием Его плоти, коей является пицца с пепперони и халапеньо, это же очевидно.
217 580566
Мб я просто не понял что он на английском говорит, НО.
Вот он говорит, что в двигателе с открытым циклом нельзя просто так взять и запихнуть выхлопы от турбины в камеру сгорания, так как пепел забьет вывод для оксислителя и топлива. Но почему бы просто не юзать топливо, что не воспроизводит пепел, как метан или водород?
алсо, я чет не понял нахуя вообще нужны водородные двигатели, учитывая что плотность низкая пиздос, в атмосфере он может ебнуть, та еще и охлаждать его нужно до 20 кельвинов?
Вот само видео, если что
https://www.youtube.com/watch?v=LbH1ZDImaI8
218 580583
>>580566

>Но почему бы просто не юзать топливо, что не воспроизводит пепел, как метан или водород?


Так и делают, о чём он и говорит. Ну а в НК-33 просто окислительный газ, и это он тоже рассказывает.

>нахуя вообще нужны водородные двигатели, учитывая что плотность низкая пиздос


https://www.youtube.com/watch?v=jI8TuufCp0M
219 580585
Посмотрел передачу про космос, где я не совсем понял. Там сказали что частица света - фото, имеет свойства частицы и волны. При этом если ты смотришь на частицу, то она ведёт себя как частица, но если ты её не видишь, то как волна - это что за бред?
И еще там сказали что у фотона есть два спина, и источник фотона испускает одновременно два фотона с одинаковыми спинами. Но при этом если ты посмотришь на один из этих фотонов, то у другого фотона спин исчезнет, даже если он был на другом конце вселенной моментально.
Я нихуя не понял, поясните пжлст.
220 580588
Допустим мы разворачиваем с поверхности Фобоса трос на многие километры и окунаем его в атмосферу Марса - таким образом можно опустить орбиту Фобоса еще сильнее?
221 580590
>>580566

>нахуя вообще нужны водородные двигатели


У тебя на картинке всё написано же, у водорода максимально возможный удельный импульс среди всех химических топлив, в плане эффективности ему равных нет. Низкая его плотность легко компенсируется увеличенным диаметром баков, глубокое охлаждение требуется любому криогенному топливу, взрываются водородные двигатели не чаще керосиновых, и т.д., все эти проблемы решаемы.

Главный недостаток водорода в том, что всё связанное с ним пиздец какое дорогое, а ракеты в современном мире в основном летают не на водороде и не на метане, а на бабле. Шиковать с водородными двигателями могут себе позволить только государственные космические агентства, где финансирование из бюджета льётся рекой и за его перерасход никто не отвечает.
222 580593
>>580395
Я обычно всегда стараюсь с расчётами пояснять. Просто не так уж часто задают вопросы, требующие расчётов.
На самом деле, хорошо, что хоть как-то задают, я больше 10 лет после школы никак не использовал физику, и забыл нафиг про джоули, законы термодинамики и прочее; а тут оно пригождается.
А вернулся я в увлекательный мир физики примерно лет пять назад когда задался глупым вопросом: почему когда идут осадки становится теплее. По той же причине, по которой мы потеем.
image.png64 Кб, 322x196
223 580594
>>580453
>>580488
Что вы несёте?

>>580451
Не слушай их. Пикрелейтед правильный ответ.
42.
224 580601
>>580585
Это невозможно понять, аналогов в макромире нет, оно не интуитивно.
Либо принимай, можешь сам убедиться с помощью экспериментов; либо ебанись на отличненько и придумай свою интерпретацию этого поведения, которая будет согласовываться с макромиром (чего никто не смог за все это время добиться); либо не заморачивайся.
Ближайшее, что ты можешь сделать к пониманию квантовой природы: научиться в научные публикации и штудировать каждую по интересующему вопросу. Тогда хотя бы будет понимание, как пришли к такой природе, которую мы рассматриваем сейчас.

>>580588
Да.
Забавный факт, если затормозить Фобос, то он будет совершать оборот вокруг Марса ещё быстрее.
225 580612
>>580590

>Шиковать с водородными двигателями могут себе позволить только государственные космические агентства


И Бизос.
226 580615
>>580612
Абажжи, разве писос не на матане летает?
227 580616
>>580601
Т.е. там верно сказали что излучатель фотонов излучает одновременно пару противоположных фотонов?
А как они между собой на бесконечном пространстве передают инфу?
Пиздос...
Почему это до сих пор не изучили?
228 580617
>>580594
Повеяло фанатом научной фантастики, пытающимся подменить пиццу с пепперони и халапеньо мемами из какого-то рассказа.
229 580620
>>580616

>Т.е. там верно сказали что излучатель фотонов излучает одновременно пару противоположных фотонов?


Нет, это в случае, когда он излучает пару противоположных фотонов. Обычно фотоны излучаются не парами.

>А как они между собой на бесконечном пространстве передают инфу?


Хуй знает.

>Почему это до сих пор не изучили?


Изучают. Думаешь, наткнулись на препятствие, забили хуй и стали в лазертег играть с киловаттными лазерными установками?
230 580639
>>580616

>передают инфу


Никакой инфы не передается, квантовая телепортация это самое худшее название, которое только можно было придумать для этого физического явления.
231 580647
Если всё вещество было создано в звёздах - чем плотнее вещемство по атомам, тем больше нужно было энергии чтобы его сплющить в это вещество, так? А какое самое плотное вещество вооббще?
232 580648
>>580615
Нет. Нью Гленн будет на метане, но только первая ступень. У Бизоса даже лунный лендер будет на водороде.
233 580650
>>580615
Да, кстати, Арианэспас тоже не государственное космическое агентство, и тоже на водороде летают.
234 580652
https://thealphacentauri.net/schedule/ откуда у индии столько запусков? У них есть свой маск? Когда запустят человека в космос?
235 580653
>>580650
Так и Боинг и прочие тоже не государственные, а подрядчики. Что с того? Без госфинансов гидроген заюзал писос только, и то не заюзал, так что тот анон был прав.
236 580654
>>580647
Нейтронная материя. Из нее нейтронные звезды состоят. Можно сказать, что вся звезда - это большой атом. Но лучше так не говорить, особенно здесь, т.к. тебя за это обоссут и обосрут.
237 580655
>>580647

>Если всё вещество было создано в звёздах


Нет, водороду звёзды не нужны.

>А какое самое плотное вещество вооббще?


Загугли ёпт. А если ты имеешь в виду самый тяжёлый стабильный элемент, то загугли ёпт. А если тебе не обязательно вещество состоящее из атомов, то нейтронное вещество, из которого состоят нейтронные звёзды.
238 580656
>>580653

>и то не заюзал


В смысле блядь, а Нью Шепард на чём летает?
239 580657
>>580654
>>580655
оганессон - оно? Поясните. Т.е. звезда должна была быть экстремальной чтобы его создать или как? Я не понимаю. Пишут что его вообще сделали в лаборатории и он газ.
240 580659
>>580652

>говносайт точка нет


Вы же в курсе, что эта ссылка не сохраняет сортировку по стране?

>откуда у индии столько запусков?


У них стабильно 5-7 пусков в год, и уже давно. Часть из списка наверняка уедет на другой год, у них даже дат нет.

>У них есть свой маск?


Нету, у них есть смекалочка.

>Когда запустят человека в космос?


Планируют в 2022. Возможно и вообще не запустят, бюджеты мизерные.
241 580660
>>580657
Да, самые тяжёлые элементы создаются в лабораториях на ускорителях. Но в космосе они тоже создаются, во взрывах сверхновых, столкновениях нейтронных звёзд и подобных процессах. Вообще-то, большая часть таблицы Менделеева создаётся именно там, а не в обычных звёздах.
242 580661
>>580583
пересмотрел и понял. Надо инглиш учить.
>>580590

>Низкая его плотность легко компенсируется увеличенным диаметром баков


Физику тоже надо учить.
Лан, спасибо за пояснения, аноны.
243 580662
Всё очень сложно.
244 580665
>>580647

>Если всё вещество было создано в звёздах


Почти весь существующий гелий и треть существующего лития были созданы прямо в первые полчаса после Большого Взрыва.
245 580666
>>580656
На керосине.
image.png254 Кб, 889x622
246 580668
>>580657
Вот хорошая картинка.
247 580670
>>580666
Нью Шепард, это не астронавт, а суборбитальная ракета Безоса.
248 580672
>>580666
На керосине только мошейники.
https://en.wikipedia.org/wiki/BE-3
249 580675
>>580670
>>580672
Че за нахуй?! Она точно на керосине летала, я абсолютно точно помню, кто-то в википедии поменял что-то?
100% на керосине была, какого хуя теперь водород пишут!??!
250 580691
>>580675
Никогда она на керосине не летала, Джефф "Вытирает-Ноги-О-Профсоюзы" Безос не стал бы шкварится об это углеродистое говно. Даже Годдард и то летал на перекиси. Ты блин на видео пусков посмотри, это похоже на керосин по-твоему?
251 580692
>>580691
Не, я не понимаю, какого хуя? Я четко помню керосин, четко помню что при посадке чадила ракета и грязная становилась, 100% так было, почему никакой инфы об этом нет? Абсолютно точно помню что был керосин, помню как свое имя и 2+2=4.
КАКОГО ХЕРА!?
image.png944 Кб, 1213x708
252 580694
>>580692
Ты не одинок. Не раз и не два бывало, что инфа, в которой я был абсолютно-мать-его-уверен, оказывалась неправдой.
Будем все вместе дружить с дядей Альцгеймером.
image.png946 Кб, 1280x720
253 580696
>>580694
Я слишком молод для альцгеймера, мне еще сорока даже нет...
254 580711
>>580659
Я и не сортировал, просто пролистай.
Даты нету и у следующего запуска старлинков, но тут именно планируемые их агенством на июнь.
255 580717
>>580657

>Пишут что его вообще сделали в лаборатории и он газ.


Думаю, во всяких взрывах сверхновых и столкновениях нейтронных он тоже создается. Просто такие элементы ебанутся как радиоактивны и разваливаются прям за мгновение. Если звезда его и создавала, тебе она об этом не скажет, он сразу же распался и никто не увидел.
256 580718
>>580717
Мне чет тоже кажется, что при сверхновых или коллизиях НЗ образуются элементы вплоть до 1000 атомного номера без проблем, если не дальше, просто существуют меньше наносекунды.
257 580736
>>580711

>именно планируемые их агенством на июнь


Вообще говоря на нормальных сайтах все их полёты стоят под грифом TBD (to be decided) или NET "не ранее июля", то есть не имеют даже примерных дат пуска - а слиплись вместе они потому, что все работы на космодромах были отложены в связи с коронавирусом. В этом месяце могут полететь только PSLV с Oceansat-3 и ещё может быть GSLV Mk II с GISAT-1.
Даже хуеплёты из альфацентаури перед своими рандомными датами поставили значок тильда, что в данном случае можно трактовать как "примерно, хуй его знает".
Не читайте советских газет, как говорится.
258 580756
>>580668
Что-то не понял про Технеций (43) и Прометий (61). Их разве не может быть во вселенной?
image.png60 Кб, 889x622
259 580760
>>580756

>во вселенной

260 580763
>>580657
>>580718
Процессы, которые происходят до конца не изучены, но вполне возможно, что там вообще ничего не распадается, так как, к примеру, воссоздать условия близкие к взрыву сверхновой, или той же нейтронной звезды просто невозможно.
261 580766
>>580760
ну ок, в солнечной системе. Но почему тогда они серым выделены?
262 580768
>>580766
Потому, что их нельзя найти в солнечной системе, хули тут непонятного?
Инб4 нашел в экспериментальном реакторе азазаз
263 580769
>>580763
Тогда какой-нибудь унбитриквадрий могли найти уже.
Но чем дальше в атомные номера, тем меньше живет атом. Пресловутый островок стабильности проскочили найдя стабильность не в миллиардные доли секунды, а в миллионные.
264 580775
Илон Маск еврей?
265 580778
NASA, например, является участником соглашения по МКС, под которым также подписались японское, европейское и канадское космические агентства. В рамках этого соглашения США несут 76,6% финансовой нагрузки, в то время как JAXA, ESA и CSA платят 12,8, 8,3 и 2,3 процента соответственно.

То есть рашка не платит?
266 580779
>>580775
Пгиличного евгея мускусом не назовут.
Британские и пенсильванские датчи в корнях у него.
267 580780
>>580778
Спроси у тех, у кого ты эти цифры взял.

>This includes NASA's budget of $58.7 billion (inflation-unadjusted) for the station from 1985 to 2015 ($72.4 billion in 2010 dollars), Russia's $12 billion, Europe's $5 billion, Japan's $5 billion, Canada's $2 billion, and the cost of 36 shuttle flights to build the station, estimated at $1.4 billion each, or $50.4 billion in total.


https://en.wikipedia.org/wiki/International_Space_Station#Cost
268 580784
>>580780
Ну так видимо после 15 все. Санкции.
fukken lold spc.jpg176 Кб, 640x400
269 580788
>>580784
Прогрессы, союзы, огурцов запускаем забесплатно, цуп и космонавты работают на чистом энтузиазме?
270 580790
>>580788
Там речь про долевое финансирование мкс.
271 580791
>>580790
Где "там"-то?
В стоимость МКС входят и миссии к ней.
272 580809
Не обоссывайте сильно за(скорее всего) платину
https://youtu.be/qUaJlu3FsrA?list=PL5BwB5wUDXk1D95Sl8-M2yuryazi8pE2d&t=1251

Почему так сильно отличались реакции на длительное нахождение в невесомости у советских космонавтов и у американцев, летавших на Джемини, почему вторые так неестественно легко(если верить галковскеру) перенесли двухнедельный сидячий полёт?
273 580810
>>580809

>почему вторые так неестественно легко(если верить галковскеру) перенесли двухнедельный сидячий полёт?


Потому что они были в джинсах.
844421db527a476ab9af6c9282b5c982.jpg792 Кб, 1920x1948
274 580813
Космонавты сидели в такой узкой херне много дней? Пиздец, дрэгоны и впрямь развитие.
>>580809
https://habr.com/ru/post/392937/ тут есть ответы
1745749919648350037318207648829531754446381n.jpg124 Кб, 640x533
275 580815
>>580813
а воду выдавал специальный пистолет порциями по 14 грамм!

http://space-horizon.ru/articles/39
276 580821
>>580763
Даже если и так, взрыв-то длится секунды. А потом вещество просто мирно разлетается, и уж тут-то нестабильные элементы ебашат радиацией и разваливаются.
277 580822
>>580813
Просто тогда Маска еще не придумали, обходились чем придется.
278 580823
>>580821

>и уж тут-то нестабильные элементы ебашат радиацией и разваливаются.


Там все элементы насыщаются нейтронами и становятся нестабильными изотопами.
https://en.wikipedia.org/wiki/Neutron_capture
279 580824
>>580823
Я к тому, что даже если там что-то и не разваливается, то только в процессе взрыва. А после него немедленно развалится, пушо условия уже вполне человеческие.
280 580832
>>580821

>взрыв-то длится секунды


До сих пор охуеваю, что миллион километров звезды за секунду схлопывается.
281 580835
>>580775
А Дмитрий Рогозин - еврей?
282 580890
Что там у блуориджина?
image.png97 Кб, 256x256
283 580891
>>580890
[Suggestive smile]
284 580918
>>580813
Спасибо!
285 580921
>>580612
Так если водород такой дорогой, то какого хуя бизос юзает его именно на ракете для космического туризма? У нее же наоборот цель максимально дешевой быть.
image.png545 Кб, 625x638
286 580923
>>580813
Не сразу разглядел, что светло-коричневое у него между ног это его рука, а не... гм... запачканный костюм.
287 580924
>>580921
Водород дешевый, три бакса за килограмм.
Самая писечка-мякотка, что нагара на движке не образует, потому и идеально для реюза подходит.
288 580927
>>580924
так анон выше написал что дорогой он
а для реюза и метан есть
289 580929
>>580927

>три бакса за килограмм


Он дешёвый, блядь.
Никто не писал, что водород дорогой.
290 580931
>>580929
>>580924
Ага, только его температура конденсации ниже чем для жидкого азота, для начала. Потом водород замечательно реагирует с металлами. Достаточно?
291 580933
>>580931
3 бакса за килограмм это уже цена жидкого.
И да, блядь, охрупчивание происходит, движки дохуя сложные, нужна йоба-криогенная аппаратура. Но водород, сука, дешевый. Если у тебя криогеника куплена и реюза на поток поставлен в идеале, то водород нихуя не будет стоить.
А после метана нагар будет оставаться, и реюз не будет получаться.
Нахуя ты дауном придуриваешься, сука? Когда речь про цену водорода - прямым текстом его цена названа, и тут, блядь, начал маняврировать, что не про сам водород говорил типа.
Уебок
292 580936
>>580924

>три бакса за килограмм


Откуда цифра? И для какого это водорода?
293 580946
>>580933

>после метана нагар будет оставаться


Откуда, если продукты реакции газообразные? Даже если что-то и осядет, то на порядки меньше чем в керосинках, которые уже реюзаются.

https://ntrs.nasa.gov/search.jsp?R=19920000461

>Neither methane nor liquefied natural gas (LNG) produces soot when burned in turbine simulator with liquid oxygen under conditions like those in gas-generator section of rocket engine

main-qimg-626258f2facf8866d797d24622378421.jpg233 Кб, 1344x2048
294 580950
Водород сам по себе не дорогой. Вообще любое топливо на фоне стоимости ракеты не дорогое. Пикрелейтед, доли разных компонентов в ступенях Атласа, топливо вместе со служебными веществами вроде газов наддува это propellants and gases. Первая ступень на керосине, вторая на водороде есличо.
Дорогой не сам водород а ракеты на водороде.
295 580956
>>580933

>3 бакса за килограмм это уже цена жидкого.


В Германии килограмм водорода стоит 10€
296 580962
>>580956
В баксах это будет 11,3 бакса за кило.
Кислород вот тут https://www.pharmacompass.com/price/liquid-oxygen пишут что по баксу за кило.
На Шатоле 106261 кило водорода и 629340 кило кислорода. Общая стоимость заправки получается 1830089 долларов. Меньше двух лямов баксов. Сто
297 580964
>>580962
Блядь. Отправилось недописанным.
Стоимость запуска Шатола 450 лямов без учета расходов на инфраструктуру. На топливо для основыных двигателей уходит примерно 0,4% от стоимости запуска. А с учетом стоимости инфраструктуры юудет 0,2% или даже меньше.
image.png142 Кб, 437x489
298 580972
>>580590
А какой прок в его эффективности, если ракета выходит огромной пиздос? Так к тому же, еще и дорогой.
Ведь если в дельте заменить водород на нормальное топливо, то грузоподьемность будет как у фалкона, если не лучше. И цена при этом существенно снизится. Нахуя эта "эффективность" нужна тогда?
299 580978
>>580964

>На топливо для основыных двигателей уходит примерно 0,4% от стоимости запуска.


На топливо в ценах каких годов? Шаттлы не летают уже 9 лет.
300 580981
>>580590

>Главный недостаток водорода в том, что всё связанное с ним пиздец какое дорогое


Главный недостаток водорода в том, что он даже в твёрдом виде крайне неплотный. В десять раз менее плотный, чем керосин. Нужно строить дилдак огромнейших размеров, иначе топливо не влезает. Поэтому у водородных ракет крайне хуёвое массовое совершенство, и этот избыток возимой массы съедает все преимущества удельного импульса.

Выйти за эти 0.08 г/см³ при хранении водорода - голубая мечта науки и техники, почти как термояд. Никакие баллоны высокого давления, никакая криогеника не помогает. Пытаются изобретать молекулярные абсорбенты, заливать водород внутрь молекул фуллерена, но пока толкового не получается нихуя. Одна надежда на металлический водород, который перевернёт в космической и некосмической отрасли вообще всё, но как его получать и как хранить - пока нихуя непонятно.
301 580983
>>580981

>Главный недостаток водорода в том, что он даже в твёрдом виде крайне неплотный


А какая плотность у металлического водорода?
302 580984
>>580981

> В десять раз менее плотный, чем керосин.


Хотя это верно, и низкая плотность водорода это проблема, но все же если рассматривать топливную пару, то разница в плотности получается не в 10 раз, а раза в 3-3,5, поскольку большую часть массы топливной пары на гидролоксе составляет кислород.
303 580985
>>580978
В сегодняшних ценах. За 9 лет инфляция не слишком большая набежала, да и ситуацию она может изменить только в сторону уменьшения доли стоимости заправки.
И в целом не критично, 0,4%, 0,35% или 0,45%. Все равно, на фоне общей стоимости запуска цена топлива крайне мала.
304 581000
Получается что люди произошли от микробов?
305 581001
>>581000
Нет, нас боженька сделал. Ну а от кого ещё?
306 581003
>>580972

>Ведь если в дельте заменить водород на нормальное топливо, то


Она от земли не оторвётся.

>А какой прок в его эффективности


В том, что выше блядь эффективность. У тебя ракета легче по массе, а дельта больше. Особенно важно для верхних ступеней.

Поэтому например пуск на ГСО на Ариан-6 будет дешевле за килограмм, чем на Флаконе-9, хотя для низких орбит наоборот.
307 581004
>>580981

>и этот избыток возимой массы съедает все преимущества удельного импульса.


Не съедает нихуя, всё равно выгодно.
308 581006
>>580964
А что тогда дорогое в ракетах?
309 581009
w480.png111 Кб, 480x363
Ионизированные топливо и окислитель. 310 581017
Посоны, а если топливо и окислитель предварительно ионизировать? Ну и у топлива отбирать как можно больше электронов, а окислитель наоборот перенасыщать ими. Усилит ли это химическую реакцию, а тем самым и эффективность двигателя?
311 581019
Либо еще какое нибудь физическое шаманство проделывать с топливом, чтобы улучшить удельную тягу. Может повышать индуктивность топлива?
>>581017
312 581027
>>581003

>не взлетит


Флакон хеви примерно такой же по размеру, но вдвое тяжелее. И вроде как летает.

> Поэтому например пуск на ГСО на Ариан-6 будет дешевле за килограмм, чем на Флаконе-9, хотя для низких орбит наоборот.


Флакон хеви на марс может доставить больше, чем ариан и дельта хеви нп гсо, вместе взятые.
313 581028
>>581017
В теории да, усилит.
На практике либо добавка от ионизации будет ничтожной, либо ты заебёшься такой электростатический потенциал удерживать. У тебя получается конденсатор размером с ракету, а плотность запасаемой энергии в конденсаторе очень маленькая.

Можно попытаться ионизировать компоненты на ходу, но энергию нужно откуда-то брать. Здесь проблемы как с электроракетными двигателями большой тяги - заебёшься столько электрических батарей возить. А если возить реактор, то проще уже сделать тормальный ЯРД.
314 581040
>>581028

>либо ты заебёшься такой электростатический потенциал удерживать


Ну только в баках и трубах это надо, а тама темпеературы то не то что в камере сгорания.
unnamed.jpg32 Кб, 512x358
315 581043
>>581028
Можно сделать омский вариант баков. В них топливо и окисл. будут выталкиваться поршнем из диэлектрика как в шприце. А регулировать поток будет гидравлический переменный дроссель. Всё из диэлектрических материалов.
15885149205662.jpg142 Кб, 1285x965
316 581048
>>581027

>Флакон хеви примерно такой же по размеру, но вдвое тяжелее. И вроде как летает.


Он совсем не такой же по размеру, по высоте примерно такой же, но диаметр Флакона 3.66 м, а Дельты 5.1 м, по объёму разница в два раза. При этом у Флакона гораздо выше тяга на первой ступени за счёт большого количества двигателей, хоть и несколько менее мощных.

>Флакон хеви на марс может доставить больше, чем ариан и дельта хеви нп гсо, вместе взятые.


Если ты имеешь в виду на ГСО с циркуляризацией, то охуеть заслуга, туда дельты больше нужно.

Если сделать ракету, которая на НОО будет доставлять столько же, сколько хевик, но при этом на водороде, то дальше она сможет доставлять гораздо большие грузы и будет оранжевой, и будет сделана Боингом. Или так, чтобы проще было: если к хевику приделать вместо его второй ступени водородную той же массы, то его дельта значительно увеличится.
317 581070
>>580950
Ну а если коротко? У маска ведь двигатели и электроника многоразовые, он что тогда вообще экономит 90+% стоимости ракеты?
318 581086
>>580946
Погодь, а какого хрена тогда никто про метан до этого века не заикался и ебашили только водород?
319 581087
>>580983

>А какая плотность у металлического водорода?


В 12-13 раз плотнее жидкого.
https://www.nasa.gov/pdf/637123main_Silvera_Presentation.pdf
320 581090
>>581086

>Погодь, а какого хрена тогда никто про метан до этого века не заикался и ебашили только водород?



Потому что среди ракетчиков есть дроч на водород - на пиковые характеристики типа ISP, на крутые материалы и так далее. В свое время в сша почти построили SSTO (X-33), но проект отменили потому что баки были не из углепластика как изначально планировали а из люминия "как у всех".

Метан к твоему сведению стал толкать Безос и Муск как коммерсы, оба сначала думали о водороде а потом дропнули (Безос частично )

// мимокрокодил
1591469433333.jpg173 Кб, 2160x1080
321 581093
Как создать силовое поле?
Допустим, есть дохуя энергии от реактора на антиматерии.
Может можно как-то распылить ионизированный металл и хитрыми конфигурациями электромагнитных полей зафиксировать его в статичном положении?

Ведь делают такими фокусами со звуковыми волнами висящие неподвижно шарики для пинг-понга.
Не вижу принципиальных ограниченной сделать так на большем масштабе.
322 581094
>>580057
Меня больше интересует, как если что его отслеживать.
На шаттле в точку лагранжа кабанчиком не метнуться.
Ой, шаттла тоже нет.

Допустим, будет какой-то мелкий устранимый дефект, что делать?
323 581096
>>580139

>Поясните, в районах северных сияний повышенный уровень радиационного фона или нет?


Да
324 581100
Объясните, почему невыгодно?

В NASA заявили, что Россия будет вынуждена пользоваться кораблями SpaceX для доставки космонавтов на орбиту.

По их словам, в агентстве планируют перестать покупать места в «Союзах», поэтому использовать только свои корабли России станет невыгодно
325 581102
>>581094
* обслуживать
326 581106
>>581100
Летать на орбиту надо часто, но людей много не надо и места будут свободны. В итоге союз будет летать полупустой все время.
Тогда становится выгоднее покупать билет на американский корабль в половине случаев, чтобы не строить каждый раз союз.
А на союзе летать, когда надо его полностью огурцами забить.

Вообще, не исключаю, что роскосмос теперь сильно скинет цены на кресло, чтобы как-то забивать союз.
Может, даже же до 10 миллионов.
Будут летать туристы и космонавты Туркменистана и ДНР.
327 581107
>>581093
Вообще - закольцевать заряженные частицы как это делают пояса ван аллена наверное можно, но без наружных катушек как в токамаке, мне кажется, будет потеря вещества.
Силовых полей как в сайфае нет и не предвидится.
328 581109
>>581094

>На шаттле в точку лагранжа кабанчиком не метнуться.


>Ой, шаттла тоже нет.


А шатал и не мог дальше НОО летать. И дело даже не в дельте.

ЖыВоСТь отслеживать будут так же как и любой другой космический аппарат. На нем антенны для этого и у наземных станций для этого слежение.

>Допустим, будет какой-то мелкий устранимый дефект, что делать?


Они пытаются продумать все возможные мелкие косяки и что может и будет выходить из строя и как это парировать.
Тройное дублирование гиродинов, компьютеров и прочего.
Если его пробуравит астероид, лепесток не раскроется или зеркало окажется опять сделанным через жопу (не окажется, после Хаббла его проверяли наверное столько же раз, сколько все остальные зеркала для телескопов вместе взятые) - то пиздос, ничего не поделать, всрали десять миллиардов долларов впустую.
329 581110
>>581106

>но людей много не надо и места будут свободны.


А раньше нужно было больше людей?

>Вообще, не исключаю, что роскосмос теперь сильно скинет цены на кресло, чтобы как-то забивать союз.


Они будут работать себе в убыток?
330 581111
>>581100
Не читай желтятину.
НАСА не планирует отказываться от Союзов, будут летать поровну то на Союзах, то на Драконах. Все от этого только выиграли, если один не полетит, то полетит второй.
331 581114
>>581110

>А раньше нужно было больше людей?


Раньше там американцы сидели и платили за это

>Они будут работать себе в убыток?


Они всю историю так работают, с подключением.
332 581118
>>581094
>>581109
Мне кажется в крайнем случае можно будет Орионом метнуться, ещё Том Хенкс потом кино снимет.
333 581120
>>581118
Орион не долетит. Он до луны расчитывается, а ЖВСТ будет сильно дальше.
334 581122
>>581100
>>581111
Покупать места перестанут, а летать на Союзах - нет. Будут меняться стульями, так сказать.

>NASA expects to end payments to Russia once the new U.S. crew ships are operational. Under the space agencies’ current plans, U.S. astronauts will continue flying on Soyuz spacecraft and Russian cosmonauts will launch and land on the new U.S. vehicles under a barter arrangement, with no funds exchanged.

335 581123
>>581120
Ближе-дальше - это неинформативная хуйня, ты по дельте скажи. Орион же может на окололунную орбиту выходить.
336 581124
>>581122
А нахуя рогозину то это?
337 581125
>>581106

>Вообще, не исключаю, что роскосмос теперь сильно скинет цены на кресло, чтобы как-то забивать союз.


>Может, даже же до 10 миллионов.


Туристы раньше летали за большие деньги, и теперь будут.
338 581126
Что будет, если в falcon 9 залить метан+кислород?водород+кислород? Гептил+АТ?
Взлетит? Взорвется? Заглохнет?
339 581127
>>581124
Чтобы хоть кому-то из наших довелось полетать бизнес-классом.
340 581128
>>581126
Заглохнет скорее всего.
341 581129
>>581123
Да. Он расчитан на то, чтобы долететь до луны и вернуться с нее.
А тут надо в 2.5 раза дальше улететь и вернуться, тут дельту считать и не надо.
342 581130
>>581123
Да хуйня твоя дельта, скажи про бочки с кислородом и дидовские поглотители CO2.
А ещё надо куда-то какать.
Если только в одиночку лететь. Вот уж миссия, достойная Джеба
343 581131
>>581125
А че перестали-то?
Денчик Тито распиздел знакомым из Блинденбургского клуба, что на станции воняет?
344 581132
>>581130
Там еще ЕВА надо проводить, оривон в ЕВА способен? И механизм швартовки.
И самое ебовое - чтобы это "что-то" что надо чинить было чинибельным.
345 581135
>>581132

>оривон в ЕВА способен?


Неа.
346 581136
>>581135
Откуда инфа?
347 581137
>>581128
А турбонасосы не распидорит из-за того что по ним непроектные виды жижи пойдут? А там уже и взрыв.
348 581138
>>581136
Из моей жопы естественно, но Орион же для ЕВА не пилился - это автобус до ЛОПГ или лунного модуля.
349 581142
>>581137
Я не технический специалист, но мне кажется, что нет. А вот наоборот если, то может. Как минимум, у керосина температура горения выше, к тому же водородные трубки оче широкие.
350 581144
>>581130
Ну понятно, его надо будет модифицировать. И например на двух человек сделать.

>>581129
Что значит не надо? Что мне твои 2.5 раза дальности должны сказать?

>>581138
А я вот нашёл что: https://www.lockheedmartin.com/content/dam/lockheed-martin/eo/photo/webt/Orion-Spacecraft-as-a-Key-Element-to-Deep-Space.pdf

>The ECLS system on Orion offers commodities for crew survival which is significant in the event that Orion is acting as a safe haven while the crew troubleshoots any problems on the DSG. Orion also has the capability for crew to perform contingency EVAs from the crew module, which allows for more flexibility in troubleshooting scenarios.

351 581145
>>581144

>Что мне твои 2.5 раза дальности должны сказать?


Что не долетит.
Screenshot2020-06-06 Missions to Lagrangian points.png24 Кб, 1473x437
352 581150
>>581145
Короче блядь я сам поискал, вот держи пикрил. Сурс: http://www.russianspaceweb.com/lagrange.html
353 581151
А может быть такая орбита, чтобы всегда над терминатором земли летать?
354 581153
>>581151
Да, называется солнечно-синхронная, очень часто используется.
355 581155
>>581153
А-а, 360 градусов в год достигаются прецессией. Теперь понятно.
356 581161
Прочитал статью, в которой написано, что "американские инженеры так и не смогли добиться таких же результатов" про спутник связи "Молния-1". Что у него был настолько мощный передатчик, что американцы тряслись в ужасе. Однако первая же загугленная статья говорит, что у первого Intelsat'а, запущенного до первой "Молнии-1", мощность передатчика была выше.
Поясните за это, я что-то неправильно понимаю?
Ещё про "Молнию-2" написано, что она использовалась для связи между ЦУПами США и СССР в полёте Союз-Аполлон, потому что у американцев не было столь же мощных спутников. Но подажжите, заявленные характеристики Intelsat'ов 4го поколения выше, чем у "Молнии-2". Неужели я снова что-то понял не так?
В общем, была ли перемога или це зрада?
357 581165
>>579564
Вопрос по похожей теме.
Реально на эффекте мейснера удержать объект размером с чемодан на восоте полуметра? Все зависит от силы магнита и количества полупроводника?
358 581182
Когда средиземное было отрезано от океана, то на глубине (как я читал) где оставалась вода, было давление 1.7 атмосфер и температура в 80 градусов Цельсия. А могло вообще там стать так жарко, что выпарение ускорялось в прогрессии и остатки моря буквально выкипали?
Алсо, как нашли огромные солевые отложения под толщами современных вод?
Алсо, какие есть сейчас ебейшие климатические условия, которых могли не видеть ранее, или не видеть в дальнейшем?
359 581195
Способны ли все производственные силы планеты захуячить ракету, способную в пилотируемые 500км/сек ну если приспичит прям? Какиъ она будет габаритов? А 1000км/с?
360 581198
>>581195
Да, технически это допустимо.
Дефайн ракета.
Потому, что брейктру камшот на бумаге может 20% скорости света дать.
И да, на бумаге им никто пока не провел по губам, т.к. они не дали конкретных деталей реализации, и фундаментально нельзя сказать, что это нереально, как если бы они сказали 120% скорости.
361 581207
>>581195
Нет, даже близко. Можешь сказать спасибо дедуле Ц, уравнение которого растёт в лютой прогрессии.

>>581198

>брейктру камшот на бумаге может 20% скорости света дать


Это не ракета, а литералли парусник, у него внешний "ветер" с земляшки дует. Ракета носит всё своё с собой.
362 581208
>>581207

>Ракета носит всё своё с собой.


Ты сейчас оставил только ССТО.
Но ладно, если не ссто, и ты вывел ракету на орбиту, то ионники тоже не ракеты.

Давай четкое определение.
ДЕФАЙН РАКЕТА.
15662097390610.jpg12 Кб, 323x323
363 581210
>>581208
Ну ты и доколебался.

>Ты сейчас оставил только ССТО.


Схуяли?

>Но ладно, если не ссто, и ты вывел ракету на орбиту, то ионники тоже не ракеты.


Схуяли? Может ты имел в виду SEP, там еще можно понатягивать сову на глобус и поманеврировать что энергия извне прилетает. Но камшот ты никак не сможешь натянуть под определение ракеты, не возит он с собой рабочее тело.

Ракета (от итал. rocchetta — маленькое веретено, через нем. Rakete или нидерл. raket) — летательный аппарат, двигающийся в пространстве за счёт действия реактивной тяги, возникающей только вследствие отброса части собственной массы (рабочего тела) аппарата и без использования вещества из окружающей среды.
364 581211
>>581210

>Ну ты и доколебался.


Ну че, блин, как я могу дать ответ на неполный вопрос?

>Схуяли?


С того, что "ракета которая носит все с собой" это ссто, не ссто выбрасывает ступень. Все.

>>Но ладно, если не ссто, и ты вывел ракету на орбиту, то ионники тоже не ракеты.


>Схуяли? Может ты имел в виду SEP, там еще можно понатягивать сову на глобус и поманеврировать что энергия извне прилетает.


Да потому, что по твоему же определению тоже внешний ветер дует. Ты четкую терминологию дай и не будет непонятнок.

>Но камшот ты никак не сможешь натянуть под определение ракеты, не возит он с собой рабочее тело.


С чего ты взял, кто мешает ему рабочее тело возить? И кто является ракетой из дальних зондов кто использовал солнечную нергию?

>


>Ракета (от итал. rocchetta — маленькое веретено, через нем. Rakete или нидерл. raket) — летательный аппарат, двигающийся в пространстве за счёт действия реактивной тяги, возникающей только вследствие отброса части собственной массы (рабочего тела) аппарата и без использования вещества из окружающей среды.

365 581222
Почему коллапс звезд в конце их жизни внезапный? ВНЕЗАПНЫЙ?
366 581223
А не может ли быть такое что темная материя это странная материя? Страпельки не рассматривали?
367 581224
>>581222
>>581223
Нахуй иди.
368 581225
>>581222
Внезапный проколлапс
369 581234
>>581122
Представляю, как астронавтом будет гореть, когда их будут отправлять не на комфортном Драконе, а в банке огурцов.
370 581235
>>581234

>астронавтом


астронавтам
фикус
371 581258
>>581211
Что ты блядь несёшь, ты хоть сам понимаешь?

>Да потому, что по твоему же определению тоже внешний ветер дует.


Какой нахуй внешний ветер? Ты дурак?

>Ты четкую терминологию дай и не будет непонятнок.


Он дал чёткую терминологию, по ней старшот не ракета, а многоступенчатая ракета и зонд с ионником - ракеты.

>С чего ты взял, кто мешает ему рабочее тело возить?


Пока что мешает проект, таскание с собой рабочего тела не предусмотрено.

>И кто является ракетой из дальних зондов кто использовал солнечную нергию?


Все, кто попадает под определение, которое ты процитировал. То есть пока что все. Солнечная энергия - это не рабочее тело.
372 581298
>>581258

>Что ты блядь несёшь, ты хоть сам понимаешь?


А ты? Подменять терминологию не надо было.

>Какой нахуй внешний ветер? Ты дурак?


Мы начали в межгалактическом пространстве из учебника физики летать, солнца не стало? Оно настолько мощный объект, что учитывается буквально в каждом без исключения космическом аппарате. Для некоторых как непосредственный источник передвижения или ориентирования.

>Он дал чёткую терминологию, по ней старшот не ракета, а многоступенчатая ракета и зонд с ионником - ракеты.


Это не общепринятая терминология, потому и пошло недопонимание. Нахуй гореть и удивляться, когда люди внезапно не живут в том же манямирке?

>Пока что мешает проект, таскание с собой рабочего тела не предусмотрено.


Конкретно у камшота может и нет. Только камшота не существует даже на бумаге, полностью реюзабельная Энергия 2 более реальная вещь, чем он.

>>И кто является ракетой из дальних зондов кто использовал солнечную нергию?


>Все, кто попадает под определение, которое ты процитировал. То есть пока что все. Солнечная энергия - это не рабочее тело.


И да, и нет.
Солнечная энергия это не рабочее тело.
Но Горизонты (иронично, что он один), Вояджеры и Пионеры при этом ракетами не являются по общепринятому определению.
Это всё равно что называть мостом автомобиль, что по нему проехал. Ракета (космического назначения) - лишь средство вывода, и отличается от полезной нагрузки.
Давайте не путаться и использовать терминологию которую используют люди уже, а не додумывать для себя свою; а когда додумываем, давайте не будем удивляться, что вас не понимают.
373 581310
>>581161
Це зрада. Интелсаты работали по 7 лет и на геостационарной орбите. Это заебись годно и удобно.
374 581329
>>581310
А какой "объем" земной поверхности охватывается с геостационарной орбиты? Если, например, мой аппарат летит на геостационарке над атлантическим океаном - насколько широк будет охват радиопередачи? Очевидно, что обе Америки будут в зоне видимости и большая часть Европы и Африки. А в каком месте закончится видимость радиосигнала Азией?
375 581339
>>581329
Нарисуй по линейке кружок радиусом 6.3см и точку на расстоянии 35.7см. И проведи касательные к окружности.

С точки на ГСО охватывается примерно 49.5% земной поверхности. Учитывая, что спутники там дублируют друг друга, неохваченной остаётся только очень малая площадь в районе полюсов.
376 581340
>>581165

>Реально на эффекте мейснера удержать объект размером с чемодан на восоте полуметра?


Из чего чемодан-то сделан? Из аэрогеля? Его можно даже на эффекте эрекции кошачьего пениса удержать.

>Все зависит от силы магнита и количества полупроводника?


В целом да. Эффект мейснера чем-то похож на архимедову силу. Важна напряжённость вектора магнитной индукции (как бы давление) и объём выталкиваемого поля. Если бы существовал сверхлёгкий сверхпроводник, можно было бы магнитный дирижабль даже для довольно слабенького магнитного поля Земли запилить.
377 581368
>>581339
Выше 81° широты спутников вообще не видно, да и выше 75° видно только небольшой кусочек орбиты, и тот очень низко над горизонтом.
378 581371
Что именно доснимали в павильоне ? Скиньте видос
379 581390
>>581371
Что хотели, то и доснимали.
Реальная посадка на луну (если ты о ней, что следует из контекста) передавалась в прямом эфире и была доступна всем с достаточным уровнем приема.
Никто в свое время не оспаривал Спутник, т.к. слышали все радиолюбители; никто в своё время не оспаривал Аполло, т.к. все страны (смотрящие в нужное направление) могли принимать этот сигнал.
380 581403
>>581298

>А ты? Подменять терминологию не надо было.


Никто её не подменял.

>Мы начали в межгалактическом пространстве из учебника физики летать, солнца не стало? Оно настолько мощный объект, что учитывается буквально в каждом без исключения космическом аппарате


Что ту блядь нахуй несёшь?

>Для некоторых как непосредственный источник передвижения или ориентирования.


Для каких оно является "непосредственным источником передвижения"? Тех, что на солнечном парусе? Ну да, вот они не ракеты, про то и речь была.

>Это не общепринятая терминология


Это именно что общепринятая терминология, дебил.

>Конкретно у камшота может и нет. Только камшота не существует даже на бумаге


И чё, к чему это вообще, ты хоть за ходом разговора следишь?

>Но Горизонты (иронично, что он один), Вояджеры и Пионеры при этом ракетами не являются по общепринятому определению.


Почему? У них ракетные двигатели. Это вот у тебя как раз какое-то особое понимание ракеты, которое ты кстати так и не привёл.

>Ракета (космического назначения) - лишь средство вывода


Это называется ракета-носитель, конкретный тип ракеты.

Реально доебался на пустом месте, с самого начала всё было понятно, ты тупо доебался, причём без всяких оснований.
>>581207

>Это не ракета, а литералли парусник, у него внешний "ветер" с земляшки дует. Ракета носит всё своё с собой.


В этом посте всё понятно, понятно, что под ракетой подразумевается аппарат с ракетным двигателем, непонимания вообще не может возникнуть, если ты не дебил.

>>581208

>Ты сейчас оставил только ССТО.


Мало того, что доёбка, так ещё и хуйня полная, ведь многоступенчатая ракета тоже всё своё носит с собой. И отстреливает ступени, которые до этого несла блядь с собой.

>Но ладно, если не ссто, и ты вывел ракету на орбиту, то ионники тоже не ракеты.


См. выше, было понятно, что имеется в виду.

>ДЕФАЙН РАКЕТА.


В следующем же посте >>581210 тебе дали определение, а ты дальше продолжил хуйню какую-то нести.

>Да потому, что по твоему же определению тоже внешний ветер дует.


Какой внешний ветер? Ты так и не написал. Вот тут как раз нихуя не ясно, что ты имеешь в виду.
380 581403
>>581298

>А ты? Подменять терминологию не надо было.


Никто её не подменял.

>Мы начали в межгалактическом пространстве из учебника физики летать, солнца не стало? Оно настолько мощный объект, что учитывается буквально в каждом без исключения космическом аппарате


Что ту блядь нахуй несёшь?

>Для некоторых как непосредственный источник передвижения или ориентирования.


Для каких оно является "непосредственным источником передвижения"? Тех, что на солнечном парусе? Ну да, вот они не ракеты, про то и речь была.

>Это не общепринятая терминология


Это именно что общепринятая терминология, дебил.

>Конкретно у камшота может и нет. Только камшота не существует даже на бумаге


И чё, к чему это вообще, ты хоть за ходом разговора следишь?

>Но Горизонты (иронично, что он один), Вояджеры и Пионеры при этом ракетами не являются по общепринятому определению.


Почему? У них ракетные двигатели. Это вот у тебя как раз какое-то особое понимание ракеты, которое ты кстати так и не привёл.

>Ракета (космического назначения) - лишь средство вывода


Это называется ракета-носитель, конкретный тип ракеты.

Реально доебался на пустом месте, с самого начала всё было понятно, ты тупо доебался, причём без всяких оснований.
>>581207

>Это не ракета, а литералли парусник, у него внешний "ветер" с земляшки дует. Ракета носит всё своё с собой.


В этом посте всё понятно, понятно, что под ракетой подразумевается аппарат с ракетным двигателем, непонимания вообще не может возникнуть, если ты не дебил.

>>581208

>Ты сейчас оставил только ССТО.


Мало того, что доёбка, так ещё и хуйня полная, ведь многоступенчатая ракета тоже всё своё носит с собой. И отстреливает ступени, которые до этого несла блядь с собой.

>Но ладно, если не ссто, и ты вывел ракету на орбиту, то ионники тоже не ракеты.


См. выше, было понятно, что имеется в виду.

>ДЕФАЙН РАКЕТА.


В следующем же посте >>581210 тебе дали определение, а ты дальше продолжил хуйню какую-то нести.

>Да потому, что по твоему же определению тоже внешний ветер дует.


Какой внешний ветер? Ты так и не написал. Вот тут как раз нихуя не ясно, что ты имеешь в виду.
381 581407
А чё это за лазеры у старлинков?
https://youtu.be/ej3IV_WW0LE
382 581408
>>581403

>Никто её не подменял.


Ракеты. >>581207

> Ракета носит всё своё с собой.


Это не определение ракеты.

>>Мы начали в межгалактическом пространстве из учебника физики летать, солнца не стало? Оно настолько мощный объект, что учитывается буквально в каждом без исключения космическом аппарате


>Что ту блядь нахуй несёшь?


То, что солнце учитывается для космических аппаратов! ВСЕГДА.

>>Для некоторых как непосредственный источник передвижения или ориентирования.


>Для каких оно является "непосредственным источником передвижения"? Тех, что на солнечном парусе? Ну да, вот они не ракеты, про то и речь была.


Никто из них не ракеты, блядь. Практически ни один из космических аппаратов НЕ ЯВЛЯЕТСЯ РАКЕТОЙ.

>>Это не общепринятая терминология


>Это именно что общепринятая терминология, дебил.


Ты дебил. Ты выдумал себе хуйню. Ты неправ.
ИЗВИНИСЬ.

>>Конкретно у камшота может и нет. Только камшота не существует даже на бумаге


>И чё, к чему это вообще, ты хоть за ходом разговора следишь?


Это ты не следишь.
ИЗВИНИСЬ, ГОВНО. дальше не читал.
383 581414
>>581408

>Ракеты. >>581207


Я вообще не ебу, что ты пытаешься сказать. И этот дебил ещё обвиняет других в том, что их трудно понимать.

>Это не определение ракеты


Ну да, тебе анон привёл определение, как только ты попросил.

>То, что солнце учитывается для космических аппаратов! ВСЕГДА.


И ЧТО? Как учитывается? Где блядь учитывается? В другой галактике не учитывается. К ЧЕМУ ТЫ ЭТО ВООБЩЕ?

>Практически ни один из космических аппаратов НЕ ЯВЛЯЕТСЯ РАКЕТОЙ.


По тому определению, которое тебе привели, практически все являются. Блядь, как же ты заебал, просто пиздец. Анон написал "ракета" в значении "аппарат с ракетным двигателем", что короче и ВСЕМ БЛЯДЬ ПОНЯТНО, КРОМЕ ТЕБЯ ЕБУЧЕГО ДЕБИЛА.

>аргументы кончились


Ок, слив засчитан.
384 581419
>>579704
да похуй насколько неверное, уснёшь от усталости в любом случае, а дальше адаптивность моска подкрутит тебе норму восприятия и ты станешь зеро-г-зависимым как остальные астро-космо навты
385 581422
>>581407
Работают на принципе "definitely not CGI I swear!"
15158625479820.jpg165 Кб, 500x500
386 581423

>когда вбросил, а аутисты начали сраться об определениях


Как типично. Прям вот как предвидел что так и будет.
387 581435
>>581390
Наши с луны голоса космонавтов передавали вроде как, или это байка.
riga-water-zorbing-1.jpg90 Кб, 660x350
388 581444
Взлетит ли марсоход на таком принципе?
389 581447
>>581444
Взлететь не взлетит, но на кочках будет знатно подпрыгивать во время бури.
390 581449
>>581447
С чего бы это?
391 581458
392 581460
>>581444
Ехать такое, конечно, будет (пока не скатится куда-нибудь). До skycrane насо так и опускало на Марс роверы - литобрейкингом, буквально бахало о поверхность. Но как ты инструменты наружу собрался выставлять?
393 581466
>>581340
Я имел ввиду сверхпроводник размером с чемодан или около того. Просто хочу замутить установку где не спичечный коробок поднимают, а что-то более внушительное. Бабки есть.
394 581468
>>581435
Нет, какой смысл передавать их с луны, когда были Молнии и Юрий Гагарин, кстати где он?

Аноны, лучшей ответьте, как сейчас задействована орбитальная группировка "Луч"? Какие задачи она выполняет сейчас?
395 581470
>>581466
Если у тебя есть бабки на поднятие чумодана, я готов вписаться на зарплату чтобы реально запилить такое несмотря на принципиальную невозможность.
396 581471
>>581468
Проверка связи перед лунной экспедицией вроде как. Была такая байка просто, про то, как американцы поймали голоса русских космонавтов с луны, а потом выяснилось, что это запись передавалась.
397 581472
>>581165
Реально. Сила магнита решает всё. Ещё в последнее время благодаря ITER появились высокотемпературные ReBCO сверхпроводники, позволяющие обходиться всего лишь жидким азотом. (и пилить компактные электромагниты с лютыми полями, в том числе)
398 581478
>>581471
Отличие баек и обс-ов от реальности в том (в космическом плане), что последнее имеет документальное подтверждение. Кто-то записывал и не один.
4chan laughing man.jpg48 Кб, 600x600
399 581479
>>581472

>благодаря ITER появились высокотемпературные ReBCO сверхпроводники

1519541167658.gif617 Кб, 320x240
400 581480
>>581479
Удачи купить ReBCO ленту тупо в магазине до ITER. До того они запиливались в полутора специализированных лабах, после этого - освоено промышленное производство.
401 581484
>>581480
"После" не равно "в следствии", друг.
Они не появились благодаря итэру, они были.
1591559723303.jpg28 Кб, 720x720
402 581493
>>581407
Это что за нахуй блять??
403 581512
>>581493
Туман от реки ранним утром над атмосферой.
404 581516
>>581407
ты серьезно думаешь увидеть луч лазера невооруженным взглядом на высоте в 600 километров?
406 581519
>>581517
“This is Zond 6. This is the Russian probe Zond 6. November the fourteenth 1968. The time is 01:52 UT. The probe is about one hour’s travel away from the Moon.”

The audio also includes a human voice speaking in Russian. A similar voice had been picked up by Jodrell Bank during the flight of Zond 5 in September 1968. It is thought to have been either a recorded message on the spacecraft, broadcast in order to test communications, or personnel on the ground relaying their voices via the spacecraft in training for a future crewed mission.
https://habr.com/ru/company/audiomania/blog/432902/
407 581522
>>581480
гдето в ЖЖ у tnenergy был пост про ВТСП, в коментах челмк что их производит (не коменты) предлагал заказать от метра.
408 581533
>>581484
Имелось в виду стали доступны для удержания "чемодана" на весу, о котором говорит анон.
Возможность просто купить - появилась именно вследствие разработки ИТЭР, было налаженно промышленное производство сверхпроводников и в частности ReBCO.

>>581522
Они даже на алиэкспрессе продаются теперь. Ленты, шайбы из YBCO.
409 581534
>>581165
Прям полуметра - охуеешь. Сантиметров 5-10 - без проблем, и не чемодан а потяжелей.
https://www.youtube.com/watch?v=bvYUq6Ox0Hc
410 581536
Новостной тред уже не поднимается?
411 581539
>>581534
Да эту поделку я видел много лет назад, неинтересно.
Хочу поднять большую платформу и высоко, типа монолита из одиссеи 2001, чтобы прохожие охуевали и падали ниц

Список покупок:
Охуенный пром электромагнит, который поднимает тачки
Сверхпроводник с алика, все что найду, на все бабки
Азот

Потом делаю каркас «монолита» из пластика , внутрь закладываю кирпичи сверхпроводящий керамики. Керамика будет в герметичных футлярах с жидким азотом, чтобы тот не испарялся. Магнит устанавливаю западлицо с землей, над ним ставлю монолит и запускаю ток.

Волнует меня только одно — как увеличение масштаба скажется на эффекте мейснера. Ну там закон куба, все такое. Вдруг для моей установки нужно будет сверхпродника купить на миллиарды долларов или магнит потребуется итеровский
412 581540
>>581536
В него просто никто не постит.
>>564993 (OP)
413 581545
Почему ультрафиолет ебически рассеивается в атмосфере из-за Рэлеевского эффекта, но лучи меньшей длины волны нет!? Как так, где логика?
414 581549
>>581545
Квантовые эффекты же. Фотончики только определенной длины волны (плюс-минус) взаимодействуют с определенными конфигурациями частиц. Точнее, с электрончиками, определенным образом связанными с определенными ядрами. Как-то так.
415 581550
>>581549

>Квантовые


Жидовские выдумки, дальше не читал.
416 581551
>>581549
Анон, ты мне никак не пояснил. Я даже могу поддвачнуть дебила, что отписался >>581550
Реально же, разницы нет, писал бы ты ответ или нет.
Релеевское рассеяние это отличная вещь, очень хорошо многое объясняет. Но мне любопытно про другие длины волн и ты мне сказал "отъебись".
417 581586
поясните почему подобную хуйню для космоса не используют?

https://www.youtube.com/watch?v=BSu-wUXbUoI
image.png202 Кб, 403x259
418 581593
>>581586
Не смотрел видео, но превью показало, что даже пытаться смотреть такое зашквар.
КПД не может быть больше 100%.
В ЭТОМ ДОМЕ МЫ ПОДЧИНЯЕМСЯ ЗАКОНАМ ТЕРМОДИНАМИКИ.
419 581596
>>581407
>>581422
что за блять CGI?
420 581599
>>581596
Обычный CGI, очевидно же.
421 581600
>>581599
Спасибо. Вот бля, а сразу не могли нормально написать? Что за люди.
422 581602
>>581539
Никак особо не скажется. Конструкцию можно сделать довольно лёгкой и пустотелой, тем более если есть магнит нормальный. Сверхпроводящая лента относительно дешевая, азота тебе в любом городе хоть цистерну нальют. Но вот с выкипанием ты охуеешь бороться, разве что несущую ленту тянуть в НЕСУЩЕМ дьюаре.

А дистанция вот скажется сильно, т.к. напряженность убывает от квадрата. Могу посоветовать впиндюрить зеркало под монолит, чтоб расстояние казалось в 2 раза больше.
423 581606
>>581593

>Не смотрел видео


но мнение имеешь. ясно.
424 581609
>>581606
Да. Когда на превью "КПД 1001%" то очевидно, что видео - мразотный блевотный кал.
425 581610
>>581609
а посмотревшим видео очевидно, что людям выделили деньги и они создали прибор, который васяны вроде тебя понять не в состоянии.
дальше что?
426 581611
>>581606
Тиртха орёт про вечный двигатель опять. Зачем это смотреть? И так всё понятно.
427 581612
>>581610
Тиртха уже от этого конкретного прибора откреститься успел, лол. Кидай посвежее что.
428 581617
>>581612
где можно посмотреть на открещивание.
image.png415 Кб, 640x480
429 581618
>>581610

>а посмотревшим видео очевидно, что людям выделили деньги и они создали прибор, который васяны вроде тебя понять не в состоянии.


Понятно лишь то, что есть тупорылые обмудки которые не шарят в простейших законах термодинамики

>дальше что?


Дальше ты идёшь на хуй, гной.
430 581619
>>581593

>даже пытаться смотреть такое зашквар


Я, прежде чем смотреть такую хуйню (если очень надо), вообще разлогиниваюсь отовсюду, ибо потом заебёшься этот шлак из рекомендаций вычищать.
431 581621
>>581618

>в простейших законах термодинамики


нихуя узколобика порвало.
небось трудишься в нии на росийскую кококосмонавтику за 17 000 с твоим образованием коридора церковно-приходской.
432 581624
>>581619
Можно просто приватный просмотр открыть.
433 581643
>>581602
Ну что же, фундаментальных ограничений нет, буду пилить летними вечерами.

Сверхпроводник конечно будет в дюаре, а тот в конструкции, нахуй мне это выкипание. Не понимаю почему так не сделали в ховерборде лексуса
434 581644
>>581643

>Сверхпроводник конечно будет в дюаре, а тот в конструкции



Точнее сама конструкция и будет Дюаром
435 581679
траектория движения меньшего тела относительно большего при слингшот эффекте - кривая второго порядка?
или тело все-таки колеблется относительно какой-то оси при движении?
436 581686
Посмотрел сериал про космос К.Сагана, оч понравилось. Сейчас смотрю в котором Нил Тайсон, еще лучше. Ну и по традиции два тупых вопроса:
1) галактики удаляются друг от друга, получается что с каждым днем все эти потенциальные миры для заселения становятся еще дальше,еще недостижимее? т.е пока все это к хуям разлетается, у нас качают нефть и не осилили до сих пор высадку человека на марс. Кажется как то все печально что нихуя мы никуда не улетим и вымрем как обычный вид любой другой.
2) ничего быстрее чем скорость света в природе не существует ? получается что даже если изобрести способ двигаться с этой скоростью, понадобится дохуятысяч лет чтобы долететь до каклй-нибудь планеты.
какая то хуйня с космосом. неужели это так и будет на уровне "слетали на луну,запустили вояджеры на том и помянем". Где русские Саганы, где астрономия в школе ( у меня была, но сказать что училке было похуй - ничего не сказать). Одно дело мечтать , другое дело видеть что происходит в реальности. Какие покорения планет когда у нас тут негры бунтуют и жрать нечего в африке.
437 581689
ШОК!

Я правильно понимаю, что учитывая тепловую смерть вселенной и растущую энтропию единственный варик выжить для разума — это найти пусть в соседнюю вселенную (многомировая интерпр.) или высшее измерение (интерстеллар)?

Если да, то будем жить, ОТЛЕГЛО!
438 581690
>>581686

>если изобрести способ двигаться с этой скоростью, понадобится дохуятысяч лет чтобы долететь до каклй-нибудь планеты


Ага, причём долетишь туда уже после тепловой смерти вселенной по времени этой планеты.
439 581691
>>581679
Не совсем понял вопрос. В системе отсчёта большого тела траектория маленького тела всегда будет кривой второго порядка.

>>581686

>1) галактики удаляются друг от друга, получается что с каждым днем все эти потенциальные миры для заселения становятся еще дальше,еще недостижимее?


Верно, и самый дальние улетают за космологический горизонт, то есть становятся принципиально недостижимыми. Но ты не переживай, это медленный процесс по сравнению со скоростью прогресса человечества.

>2) ничего быстрее чем скорость света в природе не существует ? получается что даже если изобрести способ двигаться с этой скоростью, понадобится дохуятысяч лет чтобы долететь до каклй-нибудь планеты.


Верно. Но по часам корабля пройдёт очень мало времени.

>неужели это так и будет на уровне "слетали на луну,запустили вояджеры на том и помянем".


Ну, не совсем. Заселить галактику вполне возможно, и даже с нынешними скоростями на это уйдёт всего-то порядка пары сотен миллионов лет. А доступным скоростям ещё есть куда расти. Космоопер конечно не будет.

>>581689
Ну да, как-то так.
440 581692
>>581690
Это если туда лететь квадриллионы лет, тепловая смерь ещё не скоро наступит.
441 581693
>>581602
Анон, поясни принципиальную разницу между ибэко-шайбами и ибэко-лентами.

Шайбы допустим я смогу собрать в брикет, положить его в чемодан и получить левитирующий чемодан.

А что делать с лентой? Класть в чемодан её рулоны?
442 581695
>>581686

>галактики удаляются друг от друга, получается что с каждым днем все эти потенциальные миры для заселения становятся еще дальше,еще недостижимее


Они в среднем удаляются. А например, Туманность Андромеды, весьма бодро летит к нам и через сколько-то там миллиардов лет столкнется с Млечным Путем.

>т.е пока все это к хуям разлетается, у нас качают нефть и не осилили до сих пор высадку человека на марс


За пару веков ничто никуда не улетит, скорости не те. Да и вообще, хули думать о других галактиках, когда Млечный Путь содержит миллиарды звезд? Рановато о других галактиках думать, ведь мы еще даже не омыли сапоги в озерах Титана.
Аннотация 2020-06-08 175047.jpg7 Кб, 213x242
443 581697
>>581470
>>581472

Что курить из литературы чтобы уметь примерно посчитать сколько мне надо ребко-пластин для поднятия n-ой массы на n-ую высоту? Методичка есть?
444 581698
>>581686

>галактики удаляются друг от друга, получается что с каждым днем все эти потенциальные миры для заселения становятся еще дальше,еще недостижимее?


Те галактики, которые рядом могут и приближаться, их собственные скорости могут быть существенно больше средней скорости разлета. Чем дальше от нас галактика, тем быстрее в среднем она отдаляется. Скорость растет примерно на 20км/c на каждый миллион световых лет. То есть скорость разлета в радиусе 150 миллионов световых лет не превышает 1% от скорости света, а в ближайший миллион лет расстояния вырастут не более чем на 1%. Этот радиус как раз приблизительно охватывает местное сверхскопление из ~30000 галактик.
На фоне недостижимости галактик просто по факту современных расстояний, разлет не очень заметен.

>ничего быстрее чем скорость света в природе не существует ? получается что даже если изобрести способ двигаться с этой скоростью, понадобится дохуятысяч лет чтобы долететь до каклй-нибудь планеты.


Расстояния до ближайших звезд исчисляются световыми годами, так что тысяч лет не требуется. Скорость в 20% от световой позволяет долететь до ближайшей звезды за ~21 год, и 57 ближайших звезд будут доступны за время полета менее 100 лет.
445 581699
>>581691
вопрос был в том не будет ли там на траектории наблюдаться "биений" ну или колебаний относительно центра масс малого тела.
ну там большое тело прецессирует, малое вроде тоже должно.
но наверное этого не замеряли никогда...
>>581689
есть еще третий варик - понять что такое темная энергия, как её использовать и замутить свою вселенную с блэдждеком и куртизанками, бо этой энергии во вселенной с избытком. это конечно если современные воззрения на это дело хоть сколько-нибудь верны, а то может оказаться что никакой темной энергии и нету.
446 581700
>>581698

>Скорость в 20% от световой


а когда создадут двигатели и источники энергии, которые позволят таких скоростей достичь?
447 581702
>>581700
Хз, может и никогда. Но всяко они вероятнее, чем сверхсветовые.
Daedalusship.png312 Кб, 555x474
448 581704
>>581700
Дополню еще. Корабль на 12% скорости света, Дедал, был даже просчитан по энергетике, массе и прочему еще в 70-ых.
449 581706
>>581695
Говори за себя.
450 581710
>>578517 (OP)
в будущем лунная орбитальная станция будет постоянно видна невооруженным глазом около Луны?
451 581711
>>581710
если будет размером с эверест, тогда да
452 581712
Не совсем по теме, но тем не менее. Почему отказались от сверхзвуковых лайнеров? Разве полет на сверхзвуке на высоте 20 км не экономичнее полета на дозвуке на 10 км?
453 581714
>>581704

>Дедал


как они просчитывали термояд, если его даже сейчас еще нет?
454 581719
>>581712

>Разве полет на сверхзвуке на высоте 20 км не экономичнее полета на дозвуке на 10 км?


Нет. Конкорд жрал 17 литров топлива на 100 пассажирокилометров, дозвуковые самолёты на турбовентиляторах того времени — около 4, а современные самолёты даже 2-3 литра. Сверхзвук это всегда очень дорого.
455 581720
>>581710
В далёком будущем будет. А ЛОП-Г только в телескоп как светящаяся точка, когда отражает свет прямо в телескоп.
456 581721
>>581719
А если скажем летят ВИП-персоны, у которых время стоит гораздо дороже и которые готовы переплатить за билет в 5+ раз?
457 581723
>>581700
Таких скоростей можно было бы достичь уже сейчас. Но не всё так просто. Например, можно потихоньку разгоняться ионным двигателем. Но откуда брать энергию? Вариант 1: везти с собой ядерный реактор. То есть ты добавляешь дохуя массы к своему кораблю. Плюс нужны будут радиаторы площадью с московскую область, ещё масса. Вариант 2: светить лазером с Земли. Значит, нужен очень мощный лазер. Где его размещать? Можно на Луне, но где гарантия, что ты не будешь потом его использовать в качестве оружия? Ну, и радиаторы с московскую область всё равно понадобятся, только уже на Луне. Но это всё равно гораздо проще. Вариант 3: ядерный взрыволёт. Опять же, нужно и охлаждение, и как-то решать с международным сообществом вопрос размещения тысяч термоядерных бомб в космосе. Вариант 4: вот этот Дедал >>581704, но у него опять-таки проблемы с генерацией электричества, но можно светить с Земли опять. Но с ним проще, потому что ему меньше времени на разгон нужно.

Наконец, ещё очень большая проблема в том, как пережить полёт на такой скорости через межзвёздную среду.
458 581731
>>581723
Опять эти сайфайные фантазии. Напомню что это тред тупых вопросов, а не ответов, а сайфач в другой стороне.

>Например, можно потихоньку разгоняться ионным двигателем.


У наиболее экономичных (и слабых) ионных двигателей скорость истечения не превышает ~200км/сек.
459 581732
>>581714

>как они просчитывали термояд, если его даже сейчас еще нет?


В виде бомбы (а значит и в виде взрыволёта) термояд давно есть.

>>581712

>Не совсем по теме, но тем не менее. Почему отказались от сверхзвуковых лайнеров? Разве полет на сверхзвуке на высоте 20 км не экономичнее полета на дозвуке на 10 км?


Из-за других требований к планеру в конкорд влезало значительно меньше паксов, чем в сопоставимый по размерам дозвуковой широкофюзеляжник. И это съедало весь профит.

>>581721
Рыночек показал, что таких вип-персон недостаточно.
460 581733
>>581712
От Ту-144 отказались потому, что это был имиджевый/каргокульт-аппарат без реальных задач в советском государстве. От Конкорда отказались потому, что
- он шумный, и давал слишком громкий хлопок. Переходить на сверхзвук приходилось далеко над морем, что съедало приличную часть экономии времени.
- после 9/11 сильно закрутили гайки в аэропортах, и "просто сел и полетел" стало нереальным, что окончательно добило экономию.

>>581721
В теории для того он и задумывался.
461 581734
>>581732

>Из-за других требований к планеру в конкорд влезало значительно меньше паксов, чем в сопоставимый по размерам дозвуковой широкофюзеляжник. И это съедало весь профит.


На это было вообще похуй

>Рыночек показал, что таких вип-персон недостаточно.


Как раз их было достаточно. Причины отмены конкорда - не рыночные, и перечислены здесь >>581733
462 581735
>>581721
Ну вон конкорд как раз и выяснил, что таких персон нет. Рыночек его порешал, и летал он только за счет государственной поддержки.
463 581736
>>581735
Нет, рыночек его не решал.
464 581738
>>581686

>2) ничего быстрее чем скорость света в природе не существует ?


Ничего быстроее, чем скорость света в вакууме. При этом на скорость самого вакуума ограничений нет, в районе космологического горизонта он вполне себе движется с околосветовыми скоростями (и со сверхсветовыми за космологическим горизонтом). А по последним данным вокруг компактных массивных быстровращающихся тел этот самый вакуум ещё и вращается.

Физических запретов на возможность разогнать один кусок метрики вакуума относительно другого куска метрики вакуума не существует. Как это сделать инженерно - хуй его знает, но каким-то хуем пространство участвует в гравитационном взаимодействии и этой гравитацией тянется, значит наверное можно.
465 581742
>>581714
Нет не термояда, а экономически оправданных термоядерных реакторов. Никаких проблем посчитать приблизительную энергетику, УИ и итоговую скорость тогда не было.

>Нет термояда


значит не

>мы понятия не имеем как оно действует и какие там будут параметры


а

>как нам уложится в инженерные ограничения делая реактор таких размеров, что бы не вылезти за рамки бюджета


>>581731
Ты хоть почитай, с чего обсуждение началось, наркоман. Речь не о том, что мы можем сделать прямо сейчас, а что в принципе может быть достижимо без "физика это запрещает". Ни термояд, и ЭРД с гораздо более высокой скоростью истечения физика не запрещает.
466 581743
>>581736
Да, а то что самолёты кроме правительств Британии и Франции никто не купил (буквально 0 заказов), Air France/British Airways они достались по цене 1 франк/1 фунт, а все расходы на программу были списаны государством это называется рыночек не порешал. Приватизация прибылей и национализация убытков это, видимо, и есть экономический успех.
467 581744
>>581743
Смотри что говорю я:

>в неуспехе конкорда виноваты внерыночные факторы


Смотри что слышишь ты в моих словах:

>конкорд был коммерчески успешен


Чуешь разницу?
468 581746
>>581744

>в неуспехе конкорда виноваты внерыночные факторы


Проклятое правительство запрещало пассажирам покупать билеты на конкорд!
469 581747
>>581744
Какие именно внерыночные факторы мешали Конкорду в семидесятых? Почему его никто не купил? Почему государствам пришлось полностью списать расходы на разработку и постройку самолётов?

Конкорд был экономическим провалом с самого начала, даже до ограничения уровня шума и до дорогой нефти. То, что потом добавились и другие факторы, никак не отменяет того, что Конкорд был порешан рыночком еще до начала его коммерческой эксплуатации.
470 581748
>>581746
Да нет, просто он нарушал нормы шума своим хлопком, и его зарегулировали до того, что он почти не мог использовать сверхзвук, и то только на транс-атлантике. А потом добили новыми регуляциями против террористов. Единственный действительно рыночный фактор там - развитие интернета и телеконференций, немного уменьшившее необходимость личного присутствия за океаном. Но бизнес-джеты как летали, так и летают, они не подвержены двум вышеописанным регуляциям, там именно что сел и полетел, безо всяких проверок. На них у випов денег хватает, их "рыночек" почему-то не решает, хотя они медленные и при этом ещё дороже.
hydrogen.jpg90 Кб, 612x487
471 581752
>>581714

>если его даже сейчас еще нет?


А это шо?
472 581753
>>581736>>581734
Решал. И конкорды, и 144 постоянно летали полупустыми. Шум был, но над океаном всем было похуй. Критично было только для полётов над территорией США, поэтому Boeing 2707 и закрыли. От 9/11 конкордам вообще было ни холодно, ни жарко. Угнать и пилотировать такую ебанину - задача архисложная, а стоимость рамок с металлодетекторами и кинологических отрядов на стоимость такого оверпрайснутого билета влияет чуть более чем никак. А вот отсутствие в природе толп олигархов, готовых регулярно заполнять эту золотую маршрутку - это фатально. Олигархи предпочитают летать на бизнес-джетах, где им 8 часов отсасывает персональная шлюха. Торопиться им некуда.

Поэтому конкорд стал чем-то вроде диснейленда. Не средство быстрой транспортировки, а такой аттракцион для экстремалов, на который ты три года копишь, ловишь ощущения, после чего вешаешь ачивку на стену и продолжаешь летать обычными самолётами. Какое-то время этот летучий диснейленд поддерживали в качестве национального престижа. Но в двухтысячных цены на жижу попёрли вверх, и проект окончательно прикрыли.
hqdefault.jpg6 Кб, 480x360
473 581757
>>581732

>взрыволёта


да не, просто термояд регулируемый обещают всё те же расчетчики 50 лет кряду а воз и ныне там просто.
я к тому что расчеты это хорошо всё, но в условиях, когда ты не можешь быстро посчитать задачу трех тел в общем виде, это как то аргументом не выглядит.
ну в будущем может быть но мы не доживем.
я что-то уже сомневаюсь что до термояда доживу.
emiratesa388a6-eularabiansea1701073.jpg95 Кб, 750x507
474 581761
>>581753

>И конкорды, и 144 постоянно летали полупустыми.


Да потому и летали, что сверхзвук там на полшишечки был, и преимуществ такой перелет почти не давал.

>Шум был, но над океаном всем было похуй.


Блять, конечно, только до океана ещё долететь надо. На сверхзвук выходили далеко от берега, на ощутимой части траектории был дозвук, в котором он жрал ещё больше кстати.

>Критично было только для полётов над территорией США, поэтому Boeing 2707 и закрыли.


Ага, над Европой с её шумонормами не критично.

>От 9/11 конкордам вообще было ни холодно, ни жарко. Угнать и пилотировать такую ебанину - задача архисложная, а стоимость рамок с металлодетекторами и кинологических отрядов на стоимость такого оверпрайснутого билета влияет чуть более чем никак


Ты что, совсем ебобо? Какая стоимость? Вся затея с конкордом в теории была ради экономии времени! Ты хоть раз летал сам? Нельзя просто взять и сесть на большой самолет в наши дни, ты долго маринуешься в аэропорту. Что делает любую выгоду от сокращения времени перелёта бессмысленной. В европке любые объемы полетов сократились с закручиванием гаек, не то что Конкорда.

>отсутствие в природе толп олигархов, готовых регулярно заполнять эту золотую маршрутку - это фатально. Олигархи предпочитают летать на бизнес-джетах, где им 8 часов отсасывает персональная шлюха.


Заебись, нет денег на экономию времени в комфортном большом самолёте, слабо подверженном турбулентности, но есть деньги мариноваться в кербальском вомит комете в воздушных ямах часами. "Отсасывает персональная шлюха" это бред, там тот же уровень сервиса, и стюарды точно так же от компании. Их заказывают именно потому что это экономит время на проверках в аэропортах.
Касаемо комфорта в них: https://avherald.com/h?article=4a5e80f3/0017
475 581785
>>581695

>Они в среднем удаляются. А например, Туманность Андромеды, весьма бодро летит к нам и через сколько-то там миллиардов лет столкнется с Млечным Путем.


Через 4500 мегалет.
Стикер255 Кб, 500x500
476 581793
>>581704

>Дедал


Это портманто от "дед доедал"?
477 581794
>>581735
>>581733
>>581732
А как тогда Илон найдет летателей по суборбите?
478 581795
>>581748
Конкорд сворачивать начали до 911, он тут ни при чем. Это все равно, что говорить, что 911 закрыл Шаттлы.
479 581800
у меня тут идея возникла.
возможно она покажется кому-то тупой так что не обессудьте.
представьте что на геостационарной орбите земли висит соленоид.
ось соленоида направлена параллельно касательной к экватору.
мы берем постоянный магнит и запускаем его по такой траектории относительно планеты, на которой он согласно эффекту слингшота разгонится. в точке, где его скорость будет максимальной, он влетает в этот самый соленоид.
правильно ли я понимаю, что силы возникшие в соленоиде при взаимодействии с магнитом, придадут соленоиду ускорение?
ведь можно представить, что таких магнитов будет много. еще можно представить, что они влетают друг за другом и их энергии достаточно, чтобы пролететь насквозь, не задерживаясь в соленоиде, а просто передать ему часть энергии.
по идее таким образом ведь можно разогнать любую массу, до достаточной скорости, чтобы полететь куда-нибудь дальше Луны на большой массе. без огромного запаса топлива, ну взять с собой для маневра и торможения.
объясните мне почему это не будет работать и в каком месте я затупил? ну или может это уже кто-то сделал, а я не в курсе, где почитать. спасибо.
480 581801
>>581800
Да, соленоид ускорится, третий закон Ньютона (вроде).
Ты можешь ускорить йобу на орбите, но сам с нее сойдешь из-за этого.
481 581802
>>581800
Хуйня какая-то, я не понял твою схему, но это и не важно. Если ты используешь гравитационный манёвр, чтобы разогнать магнит, чтобы потом им разогнать что-то ещё, то это глупо, потому что вместо этого можно было сразу использовать гравитационный манёвр, чтобы разогнать ПН.
482 581812
>>581801

>Ты можешь ускорить йобу на орбите, но сам с нее сойдешь из-за этого.


почему обязательно сойдешь? ведь по идее после слингшот маневра спутник будет иметь бОльшую энергию, чем до него.
задумка в том, чтобы он тратил этот избыток на увеличение энергии большого тела на орбите.
может ты затормозишься на столько, что потом вернешься, и корректировкой сделаешь еще виток.
ну короче физических препятствий нет или я и не вижу.
фактически ведь это передача энергии движения планеты объекту на орбите посредством частичного торможения об объект.
то есть по идее можно и просто разгонять абсолютно неупругим ударом, но тогда масса разгоняемого объекта будет увеличиваться.
а если управляемый спутник будет проходить насквозь, передавая энергию через электромагнитное поле, то он может быть многоразовым.
а если применить магниты последнего поколения или из сверхпроводника, то может даже и выхлоп будет заметный при передаче.
15887419194230.png119 Кб, 512x382
483 581813
>>581802

>Хуйня какая-то, я не понял твою схему, но это и не важно.


зачем ты акцентируешь внимание на том, что ты не понял?
484 581814
>>581794

>А как тогда Илон найдет летателей по суборбите?


У Илона:
1. Заявка на 30 минут в любую точку планеты, что уже на порядок превосходит возможности самолётов, многократно уделывая даже конкорд.
2. Заявка на 100$/кг на НОО, а значит 30-50$/кг на суборбитальной. Т.е. при среднем весе пассажира 100кг получаем 3-5 килобаксов за билет.

Вообще всё равно цены на уровне конкорда получаются, но по крайней мере во времени уже очень серьёзная выгода. Если мошейник найдёт способ упустить цену хотя бы до 1-1.5 килобаксов, уже можно будет рассматривать БФР земля-земля как массовый транспорт. Хотя лично мне эта концепция с самого начала видится ебанутой.
485 581818
>>581814

>в любую точку планеты


Что, прям в любую? В тайге посадит меня? Их Христиании в Силенд смогу смотаться порыбачить, и обратно?

>многократно уделывая даже конкорд.


А проверки на входе там такие же будут? Сколько часов занимать будут? Доехать до космопорта сколько минут занимает? Подготовка к пуску опять краном за 0.0001 наносекунды, как было с фалконом?

>Заявка на 100$/кг на НОО, а значит 30-50$/кг на суборбитальной. Т.е. при среднем весе пассажира 100кг получаем 3-5 килобаксов за билет.


Если учесть что с фалконом были такие же предъявы с пуском по цене топлива, то становится ясно что всё это утопично и только фанаты могут на это клевать.
486 581819
>>581814

>многократно уделывая даже конкорд


На бумаге даже я могу конкорд уделать. Раз в тыщу. Или в пять.
487 581820
>>581814
А над чем там должна трасса лежать?
488 581821
>>581814

>многократно уделывая даже конкорд


Разве что по цене, шуму, неудобству, опасности и дикости самой идеи.
489 581827
>>581813
Затем, чтобы ты в следующий раз попытался яснее выражать свои мысли. Откуда ты магнит запускаешь? В чём профит?
490 581843
>>581693
Оберни вокруг монолита кольцом.
491 581844
>>581800
Есть такая штука как электродинамический трос.
https://en.wikipedia.org/wiki/Electrodynamic_tether
Может использоваться для перемещения в пределах магнитосферы Земли, отталкиваясь от собственно неё самой. Можно поднимать орбиту, можно опускать, можно менять орбитальные параметры как хочешь. Проблема в том, что магнитное поле слабенькое, и это всё очень медленно. Чтобы держать станцию, нужна здоровенная хуибола, так что не очень жизнеспособно. Наиболее удобно - для перемещения кубсатов (у них стандарты верификации попроще, поэтому на них нельзя ставить химические движки и всё подобное что может бахнуть, чтобы не угробить основную нагрузку студенческим поделием ненароком), а также для свода отработавших спутников. (Tethers Unlimited испытывали в прошлом году дешёвую электродинамическую ленту в кубсате Prox-1, например)

На практике отталкивание от магнитосферы Земли очень часто используется в спутниках для поворота вокруг осей. Более точно для разгрузки маховиков ориентации, там стоят соленоиды.
492 581849
>>581818

>Что, прям в любую? В тайге посадит меня?


Если будет выгодно поставить там транспортный хаб - можно и в тайгу. Смысл в том, что любые дистанции типа из Сиднея в Лондон преодолеваются за смешное время.

>А проверки на входе там такие же будут? Сколько часов занимать будут?


Нахуй там многочасовые проверки? Багаж просветят рентгеном, собакен обнюхает - и вперёд. Угнать этот дилдак изнутри и перенаправить в белый дом один хуй невозможно.

>Если учесть что с фалконом были такие же предъявы с пуском по цене топлива, то становится ясно что всё это утопично и только фанаты могут на это клевать.


По цене пока действительно утопично. Но хуй знает, пусть сначала отправит прото-старшип на орбиту и обратно, оттуда уже видно будет.

>>581819
Межконтинентальная баллистическая хуйня в любом случае уделывает по скорости самолёт в 10-20 раз. Иначе она не межконтинентальная.

>>581820
А есть разница? Высоты 20-100км не заняты ничем, с самолётными коридорами оно практически не пересекается.
493 581851
>>581812
Грубо говоря ты сойдешь с орбиты.
Ты будешь терять орбитальную энергию, иначе никак.
494 581852
>>581814
Он может эти заявленные цифры обеспечить-то?
495 581853
>>581849

>Если будет выгодно поставить там транспортный хаб


В том-то и дело.

>Смысл в том, что любые дистанции типа из Сиднея в Лондон преодолеваются за смешное время.


Мда, так и представляю, как пассажиры садятся в небоскрёб, вместо подъема по трапу. После чего из Лондона взлетает йоба уровня Сатурна-5 или Н-1. Мёртвая зона 3км, в которую никто не допускается, 20x40км дальность слышимости, в точке приземления эта хуйня работает как звуковая пушка, направленная в ту сторону куда приземляется, приземляется она разумеется так же оглушительно, подвергая пассжиров блевотной невесомости и перегрузкам на входе. Летит эта йоба конечно же над городами, и в случае чего ёбнется на населенный пункт, потому что это фундаментально менее управляемый режим полёта, и обогнуть как на самолёте нельзя. В результате тебе придется платить втридорога, чтобы сначала ехать в удалённые ебеня, в которых построили площадку (а таких мест в той же европке раз-два и обчелся), точно так же шмонаться и оформляться в аэропорту, и кататься на вомит комете, а потом на том конце опять ехать.

Звучит заманчиво.

>Нахуй там многочасовые проверки?


Это ты меня спрашиваешь? Спроси тех кто устраивает театр безопасности уже без малого два десятка лет, и из-за кого сдулся рынок ближнемагистральных аэробусов.
496 581854
>>581853
Хотя какое нахуй 3км, что-то я ёбу дал. У шаттла была 14.5км запретная зона, а слышно было за 65+км в Орландо, уже хуево.
497 581855
>>581849

>А есть разница?


Конечно есть, если оно ёбнется, то может упасть в любом месте этой трассы.
498 581860
>>581795

>On 10 April 2003, Air France and British Airways simultaneously announced they would retire Concorde later that year.[203] They cited low passenger numbers following the 25 July 2000 crash, the slump in air travel following the September 11 attacks, and rising maintenance costs

499 581861
>>581860
И че, блядь? Это никак не связано с терактами.
изображение.png498 Кб, 523x371
500 581862
501 581863
>>581861
>>581862

>the slump in air travel following the September 11 attacks


Вот это никак не связано с терактами? Пиздец, похоже на этой доске либо зумерьё, которое не понимает разницы до и после 9/11, либо просто люди которые никогда этим не интересовались.
Чтобы ты понимал, как выглядел полет самолетом в девяностых. Мой дядя в 97 году вёз из еврокомандировки серверную стойку. Купил два билета, прокатил на колесиках её и сунул рядом с собой на кресло, хоть это и было против правил багажа. Всем было абсолютно похуй, просто похуй, совсем похуй, вааабще похую. В аэропорту максимум выборочный досмотр либо просветка рентгеном (и то не везде), пришёл, сел, полетел. Примерно как на поезд. В штатах люди в буквальном смысле летали на аэробусах на работу в соседний город, отсюда и само название аэробус. После 9/11 изменилось абсолютно всё, посадочные очереди возросли в разы, такие вещи как аэробусы на работу резко перестали быть ваябл, потому что летишь ты чуть, а телепаешься в разы больше. Про международные рейсы вообще говорить нечего.
502 581867
>>581863

>Вот это никак не связано с терактами? Пиздец, похоже на этой доске либо зумерьё, которое не понимает разницы до и после 9/11, либо просто люди которые никогда этим не интересовались.


Сам ты зумер, блядь.
Они лет десять до терактов собирались свернуть программу.
Это не повод, это оправдание.

>Чтобы ты понимал, как выглядел полет самолетом в девяностых.


Ты мне перескажешь мои воспоминания что ли? Я с начала 90х и летаю. Да, ни разу на конкорде не летал, уж извините, начал с Илов и Тушек.
В штатах люди в буквальном смысле летали на аэробусах на работу в соседний город, отсюда и само название аэробус. После 9/11 изменилось абсолютно всё, посадочные очереди возросли в разы, такие вещи как аэробусы на работу резко перестали быть ваябл, потому что летишь ты чуть, а телепаешься в разы больше. Про международные рейсы вообще говорить нечего.
Вот тут я и понял, что ты пиздобол.
Тебе стоило промолчать.
503 581869
>>581827
странно, что не понял ты один, а виноват я.
профит в том, что магнит ускоряется гравитационным полем земли.
весь полученный после ускорения избыток энергии он передает большому телу, которое мы хотим ускорить, а сам возвращается на траекторию с которой повторяет маневр собственного ускорения.
таким образом он тратит энергию планеты на разгон соседствующего с ней корабля. то есть топливо на борту корабля понадобится только на торможение. а может даже оно не понадобится, если учесть что корабль можно просто сбрасывать на марс, например или еще куда.
504 581870
>>581867

>пук


Кроме ад хоминем аргументов нет, чего и следовало ожидать.

>Это не повод, это оправдание.


Ну конечно, нонейму с мэйлача видней, чем самим компаниям.
505 581871
>>581869
Нет, не ускоряется. Это так не работает.
Если в твоем манямирке чудо происходите где манягнит может отталкиваться от земли, то нахуй тебе этот манягнит? Отталкивайся сам.
506 581872
>>581870

>Кроме ад хоминем аргументов нет, чего и следовало ожидать.


Нихуя себе, гондон пытается стрелки переводить. Я объективен вообще-то. Пошёл на хуй.

>Ну конечно, нонейму с мэйлача видней, чем самим компаниям.


Пошёл. На. Хуй.

Всё таки анонимность вредна. Я бы хотел тебе ебало монтировкой раскроить, чмошник ебаный, чтоб ты больше не пиздел.
Пососать хочешь.webm683 Кб, webm,
720x544, 0:02
507 581873
>>581872
Вот это подрыв объективного раскраивателя.
508 581874
>>581871

>Нет, не ускоряется.


ты отрицаешь существование слингшот эффекта?
509 581876
>>581873
Ты кал.
Никакого подрыва. Ты реально неправ, ты реально говно, ты реально чмо.
Желаю тебе смерти.
510 581877
>>581874
Нет, ты хуйню порешь, даун ебаный.
511 581878
>>581877
какой быстрый слив.
512 581879
>>581878
Пошёл на хуй, кал.
Я объективно прав, на моей стороне истина. Ты - даун и чмо. Чтоб ты сдох.
513 581880
штош.
вне зависимости от наличия непонимающих написанной русским языком дегенератов в треде, я продолжу мысль.
что если большее тело(теперь это будет не соленоид, а цилиндр собранный из набора тороидов) будет не замедлять меньшее тело, покрытое магнитной оболочкой, а ускорять его.
ведь с магнитами это достаточно просто сделать.
современные охлажденные до рабочей температуры сверхпроводники создают достаточно мощные поля.
фактически это нечто будет работать как пушка гаусса, но только разгон магнита будет осуществляться внутри магнитного поля без приложения огромных токов.
514 581881
>>581869
Ты не ответил на мои вопросы. Откуда ты запускаешь магнит и в чём профит по сравнению с обычным использованием гравитационного маневра?
15887419194230.png119 Кб, 512x382
515 581882
>>581879

>на моей стороне истина


сказал дегенерат не понимающий и переспрашивающий очевидное.
516 581883
>>581881

>Откуда ты запускаешь магнит


магнит - это спутник на орбите.
его поверхность покрыта самым мощным постоянным магнитом, который может произвести человечество на сегодняшний день.
корректирую орбиту спутника его можно разгонять за счет падения в гравитационную яму планеты.
они приобретает энергию, которую впоследствии теряет, залетая в большой соленоид и передавая ей за счет собственного торможения внутри соленоида. он пролетает соленоид навылет.
и возвращается на свою орбиту. после этого процесс повторяется.
517 581884
>>581881
профит в том, что тебе понадобится очень малое количество энергии только на маневры вокруг планеты.
топливо тратить не надо. на спутник можно поставить солнечные батареи и ионники маневровые.
а вместо процесса стыковки будет процесс прохода кольца. потом спутник возвращается на орбиту - копит энергию от солнца на маневр и повторяет цикл, пока тело большое не приобретет нужную энергию для полета. да и не только полета, может там можно поставить аккумуляторы, которые будут собирать таким образом запас в конденсаторах.
518 581885
>>581883
Лол. Гравитационные маневры работают только на пролётных траекториях.
519 581886
>>581885

>Гравитационные маневры работают только на пролётных траекториях.


хорошо хоть ты признал, что они вообще работают.
кто сказал что моя траектория не пролетная?
просто вследствие пролета тело возвращается на траекторию, где оно может повторить пролет.
если оно смогло погасить энергию, разумеется.
520 581887
>>581886

>тело возвращается на траекторию, где оно может повторить пролет


Это на какую же? С гиперболы улетело на эллипс, а дальше куда?
521 581888
>>581887
дальше собирает солнечными панелями энергию для маневровых и с него потихонечку ионниками обратно на гиперболу?
522 581890
>>581886
Ты, если что, не со мной одним тут разговаривал.

>кто сказал что моя траектория не пролетная?


Ты сказал: >>581883

>магнит - это спутник на орбите.


Или скажешь, что ты имел в виду на околосолнечной орбите?

>>581884
Где у тебя ПН? На соленоиде? Ну так тебе его надо выводить, потом ещё магнит забрасывать на пролётную траекторию, и всё это вместо того, чтобы просто использовать гравитационный маневр на своей ПН, если уж у тебя есть такая возможность.
523 581891
>>581890
чтобы использовать гравитационный маневр для большего тела его еще разогнать надо.
идея была в том, чтобы сообщать энергию телу с большой массой но без скорости, которая позволяет ему лететь далеко на орбите.
еще раз идея была в том, чтобы передавать энергию движения планеты большому телу, подталкивая его меньшим.
без сжигания топлива.
524 581892
>>581891
А ты то есть увеличиваешь скорость маленького тела, а оно волшебным образом передаёт её большому телу, в обход законов Ньютона? Если у тебя маленькое тело взаимодействует с большим, то большое приобретёт очень мало скорости, а маленькое потеряет дохуя. И ради этого ты тебе нужно ещё и значительно увеличивать массу корабля, и разгонять маленькое тело, и всю эту схему городить.
525 581894
>>581892

>в обход законов Ньютона?


оно как раз это делает с помощью 3-го закона.
размер не важен, топливо на разгон не нужно.
и переданная энергия зависит от силы сцепления магнитными полями и скорости.
ладно я понял уже, что это никто не считал даже.

так что на счет магнитного рельсотрона в космосе?
физика тоже мешает в каком-то месте?
526 581895
>>581880
Ты про магнит на орбите? >>581800 Вот этот?
Он изменит скорость после первого же "выстрела". Сколько силы он приложит к цели, столько же получит сам. Если магнит массой в тонну добавит цели массой в тонну 3 км/с, то после этого вообще вертикально упадет на землю. А если замедлит, то улетит от Земли нахуй в межпланетное пространство.
527 581897
>>581895

>Вот этот?


нет.
есть такая хуйня.
называет магнитная дорожка.
грубо говоря она создает воронку магнитную.
если сделать её в трехмерном пространстве и в неё бросать магниты, то они по идее должны ускоряться.
а она может хоть на стационарке висеть. она им не сообщает свою энергию. магниты разгоняют другой магнит.
представь что у тебя шар из постоянного магнита падает в магнитный тороид.
внутренний диаметр тороида немного больше диаметра шара.
если сделать тороид неправильным относительно оси по которой движется шар, удлинить например и сделать внутреннюю полость конусообразной, то по идее шар будет разгоняться.
что если шар - это твой полезный груз, который влетает в эту воронку и ускоряется за счет втягивания и пролетает её насквозь, приобретая скорость.
528 581903
>>581897

>она им не сообщает свою энергию. магниты разгоняют другой магнит.


И он магически к небесной тверди прибит? Если магнит "ускоряет" что-то, то он пропорционально "замедляется" сам. Кавычки, потому что разницы нет, это все одинаковое ускорение.
529 581905
>>581903

>Если магнит "ускоряет" что-то, то он пропорционально "замедляется" сам.


а если большой магнит в это время проходит точку в гравитационном маневре, благодаря которой он получает ускорение от планеты и передает эту энергию меньшему телу?
так работать должно?
530 581927
>>581905
Гравитационный маневр действует только в системе из 3+ тел. То есть если ты не рассматриваешь ситуацию, когда магнит пролетает у Юпитера и передает скорость магниту на земной ГСО, то нет.
Гипотетически возможен обмен импульсами, когда подлетающий к Земле магнит замедлится и перейдет на замкнутую орбиту, а магнит с ГСО наоборот выкинет из системы. Но общая энергия их отностельно Земли не поменяется.
В принципе можно было бы серией межпланетных гравиманевров "возить импульс", когда скажем магнит у Юпитера меняет скорость так, что бы пролететь к Земле с большим избытком скорости и передать много энергии околоземному, но это будет полным пиздецом в отношении планирования траекторий.
531 581940
>>581927
ок. спасибо.
532 581941
>>581894
Мелкобуква, ты магию используешь, блядь.
Если будешь ускорять своим ебаным магнитом, то ты его сведешь с орбиты, как это не очевидно, сука?
533 581949
>>581941
не рвись так.
мнение местных дегенератов переспрашивающих очевидное я тоже оценил по достоинству.
ты же нихуя не понял, в том, что я написал.
разжевывать для дауна я не хочу, сорян.
534 581954
>>581949
Даун только ты, мелкобуквенный даун.
Как среднюю школу закончишь - возвращайся.
535 581962
>>581852

>Он может эти заявленные цифры обеспечить-то?


А я ебу? Я не дрочу на мошейника, как на божество, я исхожу из заявленных цифр. Во внутреннюю кухню я не вмешиваюсь.

>>581853

>Мда, так и представляю


Вообще судя по презентации у мошейника все эти проблемы решаются тем, что целый плавучий космодром подгоняется на почтенную дистанцию к прибрежному городу, а людей туда-обратно возят специальные прогулочные пароходики. Чисто гипотетически театром безопасности можно заняться прямо на пароходике, пока тот плывёт, сэкономив кучу времени.
Разумеется, это снижает гибкость системы, но самые транспортно-загруженные узлы планеты по удачному совпадению являются прибрежными городами.

>>581855

>Конечно есть, если оно ёбнется, то может упасть в любом месте этой трассы.


Нет, в этом отличие баллистических объектов от аэродинамических. Баллистический объект летит по параболе. Если активный участок нормально отработал - дальше упасть инерция мешает. (Если не отработал - ракету взрывают в полёте, предварительно спасая огурцов посредством САС, которой у мошейника пока что не предусмотрено, лол.) Дальше ракета либо садится в конечном участке параболы через suicide burn, либо там же распидорашивается.
536 581965
Самая конечно большая проблема с этой хуйнёй - это перегрузки. Там же к пассажирам будут медицинские требования как к астронавтам.
537 581968
>>558259 →
Если прям щя солнце станет 3 км в диаметре с той же массой какие подводные?
>>558406 →
за счет чего она распадается и почему одна улетает а не обе падают
538 581976
>>581962

>целый плавучий космодром подгоняется на почтенную дистанцию к прибрежному городу, а людей туда-обратно возят специальные прогулочные пароходики


Яяяяясно. Какой тогда ёбаный смысл полётов за полчаса.

>пока тот плывёт, сэкономив кучу времени


Мы потратим кучу времени, тольковыиграв кучу времени. Охуенный план, тоже такой хочу!

>Нет, в этом отличие баллистических объектов от аэродинамических. Баллистический объект летит по параболе. Если активный участок нормально отработал - дальше упасть инерция мешает.


Блять лол. В любом месте этой трассы, а не дальше.
539 581982
>>581968

>Если прям щя солнце станет 3 км в диаметре с той же массой какие подводные?


Мы замерзнем.
3801a.jpg42 Кб, 300x453
540 582067
541 582214
>>581976

>Яяяяясно. Какой тогда ёбаный смысл полётов за полчаса.



Ну там не прогулочные параходики а скоростные яхты. Но это фигня.
Если ты бохатый(а иначе нахуя тебе этовсе) то просто летишь вертолетом прямо из центра города. Возможно напрямую из офиса минуя пробки.

//мимокрок
542 623018
>>578521
В жопе у астролога.
Тред утонул или удален.
Это копия, сохраненная 10 января 2021 года.

Скачать тред: только с превью, с превью и прикрепленными файлами.
Второй вариант может долго скачиваться. Файлы будут только в живых или недавно утонувших тредах. Подробнее

Если вам полезен архив М.Двача, пожертвуйте на оплату сервера.
« /spc/В начало тредаВеб-версияНастройки
/a//b//mu//s//vg/Все доски